27th April 2019 Live Test conducted by Vajiram & Ravi

Based On January and February Current Affairs

Q1. Which of the following monuments were recently declared protected and of national importance by the Archaeological Survey of ?

(1) Hathi Khana

(2) Haveli of Agha Khan

(3) Vishnu Temple, Kotali

(4) Old High Court Building, Nagpur

Select the correct answer using the code given below: a) 1 and 3 only b) 1 and 4 only c) 2, 3 and 4 only d) 1, 2, 3 and 4

Answer: d

Explanation:

According to the information provided by the Ministry of Culture to Lok Sabha, Archaeological Survey of India (ASI) has declared 6 monuments of national importance in 2018.

Following six monuments were declared protected and of national importanceby the Archaeological Survey of India (ASI) in 2018

1. The 125-year-old Old High Court Building in Nagpur, Maharashtra.

2. Hathi Khana, a Mughal-era monument in Agra.

3. Haveli of Agha Khan, a Mughal-era monument in Agra

4. The ancient Neemrana Baori in Rajasthan's Alwar district.

5. The Group of Temples at Ranipur Jharail in Odisha's Bolangir district.

6. The Vishnu Temple in Kotali, Pithoragarh district, Uttarakhand.

Uttar Pradesh (745 monuments/sites), Karnataka (506) and Tamil Nadu (413) have the highest number of ASI-maintained sites.

Source: http://vajiramias.com/current-affairs/monuments-of-national- importance/5c3525882099376f20daa8a5/

Q2. Which of the following statements is not correct about the Central Information Commission (CIC)? a) Chief Information Commissioner is appointed by the President on the recommendation of a committee consisting of Prime minister, speaker of Lok sabha, Leader of opposition in Lok Sabha and Union Home Minister. b) Chief Information Commissioner shall hold office for a term of 5 years or till he/she attains the age of 65 years. c) Chief Information Commissioner is not eligible for reappointment but Information Commissioner is eligible for appointment as CIC. d) The Commission submits an annual report to the Central Government on the implementation of the provisions of the Right to Information (RTI) Act.

Answer: a

Explanation:

The Government of India has appointed Sudhir Bhargava as the new Chief Information Commissioner.

Statement a) is incorrect: Central Information Commission (CIC) is consists of one Chief Information Commissioner and Maximum 10 Information Commissioners. They are appointed by President on the recommendation of a committee consisting of Prime minister as chairman, Leader of opposition in Lok Sabha and Union cabinet minister nominated by PM.

Statement b) is correct: Chief Information Commissioner shall hold office for a term of 5 years or till he/she attains the age of 65 years.

Statement c) is correct: He is not eligible for reappointment but IC is eligible for appointment as CIC.

Statement d) is correct: The Commission submits an annual report to the Central Government on the implementation of the provisions of the Right to Information (RTI) Act. The Central Government places this report before each House of Parliament.

Source: http://vajiramias.com/current-affairs/chief-information- commissioner/5c3526082099376f1ece9b79/

Q3. Consider the following statements with reference to the “Samwad with Students” (SwS):

(1) It is an initiative launched by the Indian Space Research Organisation (ISRO).

(2) It aims to engage youngsters across the country to capture their scientific temperament.

Which of the statements given above is/are correct? a) 1 only b) 2 only c) Both 1 and 2 d) Neither 1 nor 2

Answer: c

Explanation:

Statement (1) is correct: As part of the enhanced outreach programme of Indian Space research Organisation (ISRO), a new platform named Samwad with Students (SwS) was launched recently.

Statement (2) is correct: Through the initiative, ISRO aims to engage youngsters across the country to capture their scientific temperament. The new conversation mission will inspire students cutting across schools and colleges.

Source: http://vajiramias.com/current-affairs/samwad-with-students- sws/5c3529aa2099376f1ece9b8a/

Q4. Consider the following statements with reference to the Agri Export Zones:

(1) The concept of Agri Export Zone (AEZ) was introduced in 2001, through EXIM Policy 1997- 2001.

(2) Recently, the Central government notified an agri export zone in Varanasi.

Which of the statements given above is/are correct? a) 1 only b) 2 only c) Both 1 and 2 d) Neither 1 nor 2

Answer: a

Explanation:

Statement (1) is correct: The concept of Agri Export Zone (AEZ) was introduced in 2001, through EXIM Policy 1997-2001, to take a comprehensive look at a particular produce/product located in a contiguous area for the purpose of developing and sourcing the raw materials, their processing/packaging, leading to final exports.

Statement (2) is incorrect: It was decided that there will be no creation of new AEZs, unless there were strong and compelling reasons. No new AEZs have been set up after 2004. All the notified AEZs have completed their intended span of 5 years and have been discontinued.

Source: http://vajiramias.com/current-affairs/agri-export- zones/5c3529212099376f2119ed11/

Q5. Consider the following statements with reference to the Circular Economy:

(1) A circular economy is an economy in which all goods are produced primarily for consumption by the people living in the country.

(2) Recovery and regeneration of products and materials at the end of each service life is a fundamental aspect of the circular economy.

Which of the statements given above is/are correct? a) 1 only b) 2 only c) Both 1 and 2 d) Neither 1 nor 2

Answer: b

Explanation:

Statement (1) is incorrect: A circular economy is an economic system where products and services are traded in closed loops or ‘cycles’. A circular economy is characterized as an economy which is regenerative by design, with the aim to retain as much value as possible of products, parts and materials.

Statement (2) is correct: It is an alternative to a traditional linear economy (make, use, dispose) in which we keep resources in use for as long as possible, extract the maximum value from them whilst in use, then recover and regenerate products and materials at the end of each service life. A circular economy is important for reducing waste, driving greater resource productivity and helping in reducing the environmental impacts of production and consumption.

Recently, NITI Aayog unveiled the “Status Paper and Way Forward on Resource Efficiency & Circular Economy”.

Source: http://vajiramias.com/current-affairs/circular- economy/5c47fa8c209937286d5301e3/

Q6. Consider the following statements with reference to the space probe “New Horizons”:

(1) It is an interplanetary space probe that was launched as a part of European Space Agency’s New Frontiers program.

(2) Recently, it flew past Ultima Thule, the most distant world ever studied by humankind.

Which of the statements given above is/are correct? a) 1 only b) 2 only c) Both 1 and 2 d) Neither 1 nor 2

Answer: b

Explanation:

Statement (1) is incorrect: New Horizons is an interplanetary space probe that was launched as a part of NASA’s New Frontiers program.

Statement (2) is correct: Recently, it flew past the most distant world ever studied by humankind, Ultima Thule, a frozen relic of the early solar system that could reveal how planets formed. New Horizons on New Year's day came within 2,200 miles (3,500 km) of Ultima Thule, which represents a pristine time capsule dating to the birth of the solar system. The fly-by marked the farthest close encounter of an object within our solar system. Since then, the probe has sent images revealing Ultima Thule to be a "contact binary" - two bodies that formed separately and then got stuck together. The formation, resembling a red-hued snowman - caused by irradiated ice - is just over 21 miles (34 km) long.

Source: http://vajiramias.com/current-affairs/ultima-thule/5c3529822099376f2119ed13/

Q7. With respect to “Rashtriya Yuva Sashaktikaran Karyakram”, consider the following statements:

(1) It is a Centrally Sponsored Scheme of the Ministry of Skill Development and Entrepreneurship.

(2) The Scheme beneficiaries are the youth in the age-group of 15-35 years, in line with the definition of ‘youth’ in the National Youth Policy, 2014.

Which of the statements given above is/are correct? a) 1 only b) 2 only c) Both 1 and 2 d) Neither 1 nor 2

Answer: d

Explanation:

Statement (1) is incorrect: The Union Cabinet has approved the continuation of Rastriya Yuva Sashaktikaran Karyakram Scheme for the Period 2017-18 to 2019-2020 with budget outlay of Rs.1160 crore. The Rashtriya Yuva Sashaktikaran Karyakram Scheme is an ongoing Central Sector Scheme of the Ministry of Youth Affairs & Sports and has been continuing since 12th Five Year Plan. The Scheme aims to develop the personality and leadership qualities of the youth and to engage them in nation building activities.

Statement (2) is incorrect: The Scheme beneficiaries are the youth in the age-group of 15-29 years, in line with the definition of ‘youth’ in the National Youth Policy, 2014. In case of programme components specifically meant for the adolescents, the age-group is 10- 19 years.

Source: http://vajiramias.com/current-affairs/rastriya-yuva-sashaktikaran- karyakram/5c352af52099376f1ece9b90/

Q8. Consider the following statements with reference to the Aadhaar and other laws (Amendment Bill) 2018:

(1) It allows ‘offline verification’ of an individual’s Aadhaar ID, without authentication, through modes specified by the Unique Identification Authority of India (UIDAI).

(2) It creates a UIDAI Fund to be used for expenses of the UIDAI, including salaries and allowances of its employees.

Which of the statements given above is/are correct? a) 1 only b) 2 only c) Both 1 and 2 d) Neither 1 nor 2

Answer: c

Explanation:

Statement (2) is correct: The Lok Sabha passed Aadhaar and other laws (Amendment Bill) 2018. The Aadhaar and other laws (Amendment Bill) 2018 seeks to amend the Aadhaar Act, 2016, the Indian Telegraph Act, 1885, and the Prevention of Money Laundering Act, 2002. It provides for ‘voluntary’ sharing of the 12-digit identification number for obtaining new mobile phone connections and opening bank accounts. It creates a UIDAI Fund to be used for expenses of the UIDAI, including salaries and allowances of its employees.

Statement (1) is correct: It allows ‘offline verification’ of an individual’s Aadhaar ID, without authentication, through modes specified by the Unique Identification Authority of India (UIDAI) by regulations.

Source: http://vajiramias.com/current-affairs/aadhaar-and-other-laws-amendment-bill- 2018/5c35328d2099376f2119ed32/

Q9. Which of the following pairs are correctly matched?

Select the correct answer using the code given below: a) 1, 2 and 3 only b) 1 and 3 only c) 2 and 3 only d) None of the above

Answer: d

Explanation:

Pair (1) is not correctly matched: Prime Minister Narendra Modi, during his visit to Baripada, in Odisha inaugurated and launched various development works. During his visit, he unveiled a digital plaque to mark the commencement of work for conservation and development of Rasika Raya Temple, and Excavated Structure at Ancient Fort Haripurgarh. In 1400 AD, Maharaja Harihar Bhanj, founded the earlier capital of Mayurbhanj State, Haripur, which is 16 km from Baripada. The ruins of the temples and palaces built then at Haripur can be found today. Among the ruins, the Rasikaraya temple and the Durbar hall of the Bhanj kings are the most significant.

Pair (2) is not correctly matched: Konark Sun Temple is located on the coastline of Odisha in Puri district. It was built in the 13th century. The temple is attributed to king Narasingha deva I of the Eastern Ganga Dynasty about 1250 CE. Also called the Surya Devalaya, the temple is dedicated to the Hindu god Surya. The temple complex has the appearance of a 100-foot high solar chariot, with 24 wheels and pulled by six horses, all carved from stone. The temple, built from Khondalite rocks, was originally constructed at the mouth of the river Chandrabhaga, but the waterline has receded since then. The wheels of the temple are sundials, which can be used to calculate time accurately to a minute. This temple was also known as 'BLACK PAGODA' due to its dark colour and used as a navigational landmark by ancient sailors to Odisha. It was given the status of a World Heritage Site in 1984 by UNESCO.

Pair (3) is not correctly matched: Jagatpita Brahma Mandir situated at Pushkar is one of very few existing temples dedicated to the Hindu creator-god Brahma in India and remains the most prominent among them. Days of construction of this temple dates back to the 14th century. The Pushkar Fair, renowned for being one of the world’s largest cattle fair, is being held near Rajasthan’s Ajmer.

Source: http://vajiramias.com/current-affairs/asika-raya-temple-haripurgarh- fort/5c35351e2099376f20daa8dc/

Q10. Agasthyakoodam, is the second highest peak in Kerala is located in which of the following Wildlife Sanctuaries? a) Neyyar Wildlife Sanctuary b) Chinnar Wildlife Sanctuary c) Wayanad Wildlife Sanctuary d) Idukki Wildlife Sanctuary

Answer: a

Explanation:

Agasthyarkoodam is a peak within Neyyar Wildlife Sanctuary, Kerala. This peak is a part of the Agasthyamala Biosphere Reserve which lies on the border between the Indian states of Kerala and Tamil Nadu. Agasthyarkoodam is the second highest peak in the State at 1,868 m. Agasthyarkoodam is a pilgrimage centre for devotees of the Hindu sage Agastya, who is considered to be one of the seven rishis (Saptarishi) of Hindu Puranas. Hence, option a) is the correct answer.

Source: http://vajiramias.com/current- affairs/agasthyarkoodam/5c3534f02099376f1ece9bbc/

Q11. With respect to “Dark Matter”, consider the following statements:

(1) It interacts with the electromagnetic force and can be detected only through its gravitational effects.

(2) Recently scientists have found the first observational evidence for the effect known as ‘dark matter heating’.

Which of the statements given above is/are correct?

a) 1 only b) 2 only c) Both 1 and 2 d) Neither 1 nor 2

Answer: b

Explanation:

Statement (2) is correct: Scientists have found the first observational evidence for the effect known as ‘dark matter heating’ i.e. the dark matter can be heated up and moved around, as a result of star formation in galaxies. The finding offers clues as to what makes up dark matter.

Statement (1) is incorrect: It is called so because unlike normal matter (i.e. stars and galaxies), dark matter does not interact with the electromagnetic force. This means it does not absorb, reflect or emit light or electromagnetic radiation of any kind. And this makes it extremely hard to spot/detect. It can be detected only through its gravitational effects.

Source:http://vajiramias.com/current-affairs/dark matter/5c3538882099376f1ece9bcc/

Q12. Consider the following statements with reference to the Open Acreage Licensing Policy (OALP):

(1) It is a policy adopted by the Government of India in 1997 indicating the new contractual and fiscal model for award of hydrocarbon acreages towards exploration and production (E&P).

(2) Under OALP, oil and gas acreages will be available round the year instead of cyclic bidding rounds as in New Exploration Licensing Policy.

Which of the statements given above is/are correct? a) 1 only b) 2 only c) Both 1 and 2 d) Neither 1 nor 2

Answer: b

Explanation:

Recently, Union Minister of Petroleum and Natural Gas has launched NIO and MRSC for Open Acreage Licensing Programme (OALP) Bid Round-II.

Statement (1) is incorrect: The Open Acreage Licensing Programme (OALP) along with the National Data Repository (NDR) were launched in June 2017. OALP envisages six monthly periodic bidding rounds starting from July 1, 2017.

Statement (2) is correct: Under OALP, oil and gas acreages will be available round the year instead of cyclic bidding rounds as in New Exploration Licensing Policy.

Source: http://vajiramias.com/current-affairs/oalp-bid-round- ii/5c3d85412099370eb9577022/

Q13. With respect to “Central Administrative Tribunal (CAT)”, consider the following statements:

1. It had been established under Article 323 - A of the Constitution of India.

2. The Tribunal is guided by the principles of natural justice in deciding cases and is not bound by the procedure, prescribed by the Civil Procedure Code.

Which of the statements given above is/are correct? a) 1 only b) 2 only c) Both 1 and 2 d) Neither 1 nor 2

Answer: c

Explanation:

According to a report by Parliamentary Standing Committee, the Central Administrative Tribunal (CAT) lacks human as well as physical infrastructure, which is leading to backlog of cases.

Statement (1) is correct: The Central Administrative Tribunal (CAT) had been established under Article 323 - A of the Constitution. To adjudicate disputes and complaints with respect to recruitment and conditions of service of persons appointed to public services and posts in connection with the affairs of the Union or other authorities under the control of the Government.

Statement (2) is correct: The Tribunal is guided by the principles of natural justice in deciding cases and is not bound by the procedure, prescribed by the Civil Procedure Code.

Salaries and Allowances and Conditions of Service of the officers and other employees of the Tribunal are specified by the Central Government. There are 17 Benches and 21 Circuit Benches in the Central Administrative Tribunal all over India. In addition the Central Administrative Tribunal, Principal Bench is dealing with the matters of Govt. of National Capital Territory of Delhi.

Source: http://vajiramias.com/current-affairs/central-administrative-tribunal- cat/5c3537222099376f20daa8e7/

Q14. Arrange the following ports from west to east:

(1) Chabahar

(2) Gwadar

(3) Colombo

(4) Sabang

Select the correct answer using the code given below: a) 1-2-3-4 b) 2-3-4-1 c) 1-2-4-3 d) 2-1-3-4

Answer: a

Explanation:

The correct sequence is Chabahar- Gwadar- Colombo- Sabang.

CHABAHAR, IRAN: This one is close to Pakistan's Gwadar port where China has invested heavily. It also falls on the North-South Transport Corridor (NSTC) that links India to Central Asia and Europe

India has taken over operations of the strategic Chabahar port in Iran. Chabahar Port project consists of two separate ports named Shahid Kalantari and Shahid Beheshti, each of which has five berths. It gives India a foothold in the western Arabian Sea, which is important as many of its energy imports pass through the route.

SABANG, INDONESIA: Its position at the mouth of Malacca Strait makes it crucial. Malacca Strait is the main shipping lane between Indian Ocean and Pacific Ocean and one of the busiest sea routes. The deep sea port (deep enough for submarines too) will help counter China's rising maritime influence in the region. Recently, during Prime Minister Modi’s Indonesia visit, India got the military access to the country’s Sabang port.

DUQM, OMAN: India gained military access to the port on Oman’s southern coast this year. Duqm, on the northwestern edge of Indian Ocean, provides easy access to Red Sea.

ASSUMPTION ISLAND, Seychelles: On paper, this is purely an infrastructure project. The agreement had to be revised due to political opposition in Seychelles but broadly it signals a step forward for India’s strategic interests.

The Port of Colombo is the largest and busiest port in Sri Lanka. Located in Colombo, on the southwestern shores on the Kelani River, it serves as an important terminal in Asia due to its strategic location in the Indian Ocean. Hence, option a) is the correct answer.

Source: http://vajiramias.com/current-affairs/chabahar-port/5c3d85392099370eba75f3c0/

Q15. With respect to “Jawahar Navodaya Vidyalaya (JNV)”, consider the following statements:

(1) They are fully residential schools for talented rural and urban children.

(2) These schools are managed and run by the Navodaya Vidyalaya Samiti, an autonomous organization under the Ministry of Human Resource Development (HRD).

Which of the statements given above is/are correct? a) 1 only b) 2 only c) Both 1 and 2 d) Neither 1 nor 2

Answer: b

Explanation:

Statement (1) is incorrect: Union Minister of Human Resource Development (HRD) announced the approval of an increase of 5000 seats in Jawahar Navodaya Vidyalaya (JNV) for the Academic Year 2019- 20.They are fully residential schools for talented rural children.

Statement (2) is correct: Navodaya is the only education system in the country where students give entrance examination for admission into Class 6.These are managed and run by the Navodaya Vidyalaya Samiti, an autonomous organization under the Ministry of Human Resource Development (HRD).

Source: http://vajiramias.com/current-affairs/jawahar-navodaya-vidyalayas- jnv/5c3d85352099370eba75f3be/

Q16. With respect to “National Cadet Corps (NCC)”, consider the following statements:

(1) It is an organization under the Central Armed Police Forces (CAPF), engaged in grooming the youth into disciplined and patriotic citizens.

(2) It is a voluntary organization which recruits cadets from high schools, colleges and universities all over India.

(3) Union Home Ministry deals with NCC at the National level and the State level.

Which of the statements given above are not correct? a) 1 and 2 only b) 2 and 3 only c) 1 and 3 only d) 1, 2 and 3

Answer: c

Explanation:

Recently Vice President Shri Venkaiah Naidu formally inaugurated National Cadet Corps (NCC) Republic Day Camp 2019 at Delhi Cantt.

Statement (1) is incorrect: National Cadet Corps (NCC) is a Tri-Services Organization, comprising the Army, Navy and Air Force, engaged in grooming the youth into disciplined and patriotic citizens.

Statement (2) is correct: It is a voluntary organization which recruits cadets from high schools, colleges and universities all over India.

Statement (3) is incorrect: Defence Ministry deals with NCC at the National level. Education Ministry deals with NCC in all States. Directorate General NCC is the national level HQ located at New Delhi. It is headed by the Director General (DG), an Army Officer of the rank of Lieutenant General.

Source: http://vajiramias.com/current-affairs/national-cadet-corps- ncc/5c3d85442099370eb7c8329d/

Q17. With respect to “National Anti- Profiteering Authority (NAA)”, consider the following statements:

(1) It is headed by a Union Minister of State, in-charge of Revenue or Finance.

(2) NAA is the institutional mechanism under GST law to ensure that customers get the full benefit of tax cuts.

(3) It will cease to exist two years after the Chairman takes charge, unless the GST Council recommends otherwise.

Which of the statements given above is/are not correct?

a) 1 only b) 2 and 3 only c) 1 and 3 only d) 1, 2 and 3

Answer: a

Explanation:

Union Minister of State for Finance informed Rajya Sabha about the National Anti- Profiteering Authority (NAA).

Statement (2) is correct: It has been constituted under Section 171 of the Central Goods and Services Tax Act, 2017 as an institutional mechanism to check the unfair profit-making activities by the trading community. Its objective is to ensure that any reduction in GST rates is passed on to the consumers by a commensurate reduction in the price of goods and services.

Statement (1) is incorrect: The NAA will be headed by a senior officer of the level of Secretary to the Government of India. There will be four Technical Members from the Centre and/or the States.

Statement (3) is correct: The NAPA will cease to exist two years after the Chairman takes charge, unless the GST Council recommends otherwise.

Source: http://vajiramias.com/current-affairs/national-anti-profiteering-authority- naa/5c3d87e42099370eba75f3cb/

Q18. The 103rd Constitution Amendment Act 2019, which was in news recently, is related to which of the following? a) To facilitate acquisition of citizenship by six identified minority communities. b) To provide reservation in public employment and higher education for economically weaker sections. c) To grant ST status to Bodo and Kacharis in hill districts of Assam and Karbis in the rest of Assam. d) None of the above

Answer: b

Explanation:

The President of India has given assent to the Constitution (103rd Amendment) Act, 2019. The Constitution (103rd Amendment) act, aims to provide reservation in public employment and higher education for economically weaker sections. Economic reservation in jobs and education is proposed to be provided by inserting clause (6) in Articles 15 and 16 of the Constitution.

Article 15(6) enables states to make reservation for any economically weaker section of citizens in any educational institution, including private institutions, whether aided or unaided, except minority educational institutions covered under Article 30(1). The upper limit of reservation will be ten percent, which will be in addition to the existing reservations. Article 16(6) enables State to make provision for reservation in jobs appointments, in addition to the existing reservations, subject to a maximum of ten percent. Hence, option b) is the correct answer.

Source: http://vajiramias.com/current-affairs/constitution-103rd-amendment-act- 2019/5c3d942a20993715cf9968bd/

Q19. What is ‘HD 21749b’, which was in the news recently? a) Exoplanet b) Advanced Air Defence (AAD) Missile of USA c) Asteroid of the Apollo group d) None of the above

Answer: a

Explanation:

NASA’s Transiting Exoplanet Survey Satellite (TESS) has discovered a third small planet outside our solar system since its launch in April last year. The new planet named HD 21749b, orbits a bright, nearby dwarf star about 53 light years away in the constellation Reticulum. HD 21749b journeys around its star in 36 days, compared to the two other planets discovered by Transiting Exoplanet Survey Satellite (TESS) – Pi Mensae b, a ‘super-Earth’ with a 6.3-day orbit, and LHS 3844b, a rocky world that speeds around its star in just 11 hours. The planet is about three times the size of Earth, which puts it in the category of a ‘sub-Neptune’. It’s more likely made of gas, of a kind that is much denser than the atmospheres of either Neptune or Uranus. Hence, option a) is the correct answer.

Source: http://vajiramias.com/current-affairs/hd-21749b/5c3d87e82099370eba75f3cd/

Q20. Consider the following statements with reference to the tokenisation service:

(1) Tokenisation involves a process in which a unique token masks sensitive card details.

(2) The RBI has allowed card payment networks to offer card tokenisation services to any token requestor, that is, a third party app provider.

Which of the statements given above is/are correct? a) 1 only b) 2 only c) Both 1 and 2 d) Neither 1 nor 2

Answer: c

Explanation:

Recently, the Reserve of India (RBI) has clarified that card payment networks cannot impose any charge on customer for availing the tokenisation service.

Statement (1) is correct: Tokenisation involves a process in which a unique token masks sensitive card details.

Statement (2) is correct: The token is then used to perform card transactions in contact-less mode at Point Of Sale (POS) terminals, Quick Response (QR) code payments, etc. The RBI has allowed card payment networks to offer card tokenisation services to any token requestor, that is, a third party app provider.

Source: http://vajiramias.com/current-affairs/tokenisation- service/5c3d87df2099370eba75f3c9/

Q21. With respect to “Web Wonder Women Campaign”, consider the following statements:

(1) It aims to recognise ‘100 Women Achievers’ in India who have excelled in diverse sectors of public work.

(2) Union Ministry of Women and Child Development has partnered with Breakthrough and Twitter India for this campaign.

Which of the statements given above is/are correct?

a) 1 only b) 2 only c) Both 1 and 2 d) Neither 1 nor 2

Answer: b

Explanation:

Statement (1) is incorrect: The Union Ministry of Women and Child Development has launched an online campaign, ‘#www:Web Wonder Women’. The Campaign aims to discover and celebrate the exceptional achievements of India women across the globe, who have used the power of social media to run positive & niche campaigns to steer a change in society.

Statement (2) is correct: For this campaign, Union Ministry of Women and Child Development has partnered with Breakthrough and Twitter India. The Campaign invites Entries via Nominations from across the world. Indian-origin women, working or settled anywhere in the world, are eligible for nomination. The shortlisted entries will be open for public voting on Twitter and the finalists will be selected by a specialized panel of judges.

Source: http://vajiramias.com/current-affairs/web-wonder-women- campaign/5c3d8b142099370eb7c832ad/

Q22. ‘Future of Consumption in Fast-Growth Consumer Market – India’, was prepared by: a) World Bank b) United Nations Educational, Scientific and Cultural Organization (UNESCO) c) World Economic Forum (WEF) d) International Labour Organization (ILO)

Answer: c

Explanation:

World Economic Forum (WEF) has released a report titled ‘Future of Consumption in Fast- Growth Consumer Market – India’. After studying China in 2017, for 2018 it turned its attention to India. Highlights of the report are India is set to become the world’s third largest consumer market behind only USA and China by 2030. By 2030, more than 40% of all purchases will be highly digitally influenced, up from 20-22% today. Over the next decade, consumption in fast-

growth consumer markets such as China, India and Southeast Asia will be reshaped by the Fourth Industrial Revolution. Hence, option c) is the correct answer.

Source: http://vajiramias.com/current-affairs/future-of-consumption-in- india/5c3d8b182099370eb7c832af/

Q23. The main objective of the recently launched Gangajal Project is to provide better and more assured water supply in which of the following cities? a) Varanasi b) Agra c) Prayagraj d) Kanpur

Answer: b

Explanation:

Prime Minister Narendra Modi launched a series of development projects worth Rs. 2900 Crores for the Agra city and the adjoining areas including dedicating the Gangajal project. Objective of the Gangajal Project is to provide better and more assured water supply in Agra. Gangajal project aims to bring 140 cusecs of Ganga water to Agra from Palra headworks in Bulandshahr's Upper Ganga canal. This will help meet the drinking water demands in the city. Agra is mainly dependent on the Yamuna River for water. Yamuna water, being highly polluted with toxins, needs to be diluted by Ganga water. Hence, option b) is the correct answer.

Source: http://vajiramias.com/current-affairs/gangajal- project/5c3d8b1b2099370eb9577037/

Q24. Glanders, a fatal infectious disease of equines including horses, donkeys and mules, is caused by: a) Virus b) Bacteria c) Protozoa d) Fungus

Answer: b

Explanation:

Glanders is a fatal infectious disease of equines including horses, donkeys and mules. The disease is caused by a bacterium known as Burkholderia mallei and has zoonotic potential. The organism is also considered as potential bio-weapon and categorized under ‘Tier 1 Select Agent’. Recently, Union Minister of Agriculture & Farmers Welfare released recombinant Enzyme-linked immune sorbent assay (ELISA) kits for two diseases: one for Glanders and other for Equine Infectious Anaemia (EIA). The ELISA is an immunological assay (analytical procedure) commonly used to measure antibodies, antigens, proteins and glycoproteins in biological samples. Hence, option b) is the correct answer.

Source: http://vajiramias.com/current-affairs/elisa/5c3d8b162099370eb9577035/

Q25. Consider the following statements with reference to the “Persian Gulf Regional Dialogue Forum”:

(1) It is an inter-governmental security forum held annually by Iran.

(2) It consists of all states of the Persian Gulf and has been advancing global peace and security since 1958.

Which of the statements given above is/are correct? a) 1 only b) 2 only c) Both 1 and 2 d) Neither 1 nor 2

Answer: d

Explanation:

Statement (1) is incorrect: Addressing the 2019 Raisina Dialogue Conference in New Delhi, Iran Foreign Minister Mohammad Javad Zarif proposed the formation of a “Persian Gulf Regional Dialogue Forum”.

Statement (2) is incorrect: It is a proposed diplomatic initiative to address the lingering conflicts and mistrust in the Gulf region. The proposed forum can look into the inviolability of international boundaries; Non-interference in internal affairs of others; the peaceful settlement of disputes; the unacceptability of threats or use of force, and the promotion of peace and prosperity in the region.

Source: http://vajiramias.com/current-affairs/persian-gulf-regional-dialogue- forum/5c3d8b132099370eb9577033/

Q26. With respect to “Sino-Indian Digital Collaboration Plaza (SIDCOP)”, consider the following statements:

(1) Its objective is to bring Indian IT companies and Chinese enterprises closer to each other on a single AI enabled platform.

(2) It is a partnership between NASSCOM and China’s Ministry of Industry and Information Technology.

Which of the statements given above is/are correct? a) 1 only b) 2 only c) Both 1 and 2 d) Neither 1 nor 2

Answer: a

Explanation:

Statement (1) is correct: The Sino-Indian Digital Collaboration Plaza (SIDCOP), an initiative to bring Indian IT companies and Chinese enterprises closer to each other on a single AI enabled platform was launched on 10th January 2019.

Statement (2) is incorrect: It is a partnership by National Association of Software and Services Companies (NASSCOM) with Municipal Governments of Guiyang and Dalian. SIDCOP, which is a boundary-less marketplace, will connect top providers from India with Chinese enterprises to source the right solution providers for their projects.

Source: http://vajiramias.com/current-affairs/sino-indian-digital-collaboration-plaza- sidcop/5c3d8fcc2099370eba75f3dd/

Q27. With respect to “Sanskriti Kumbh”, consider the following statements:

(1) It is a cultural extravaganza aimed at encouraging creative writing in Sanskrit.

(2) It is organized by the Government of Uttar Pradesh.

Which of the statements given above is/are correct?

a) 1 only b) 2 only c) Both 1 and 2 d) Neither 1 nor 2

Answer: d

Explanation:

Statement (1) is incorrect: Sanskriti Kumbh is a 29 days Cultural extravaganza being held at Kumbh Mela Area, Prayagraj, Uttar Pradesh. Its objective is to showcase the rich Cultural Heritage of India in all its rich and varied dimensions viz. Performing Arts- Folk, Tribal and Classical art forms, Handicrafts, Cuisines, Exhibitions etc. all in one place.

Statement (2) is incorrect: The Ministry of Culture, Govt. of India is organizing it. Under it, cultural performances of dance, music and folk art have been held for 29 days. Rashtriya Shilp Mela is being organized where Shilp Haats are set up to display handicrafts and live demonstration of the skill by craftsmen. Recently, it was inaugurated by the Governor of Uttar Pradesh in the presence of the Union Minister of state for Culture.

Source: http://vajiramias.com/current-affairs/sanskriti-kumbh/5c3d8fca2099370eb7c832b2/

Q28. Consider the following statements with reference to the National Clean Air Programme (NCAP):

(1) The programme’s objective is to put in place mitigation actions for prevention, control and abatement of air pollution only in places where real time PM 2.5 Air Quality Index (AQI) is severe.

(2) It aims to reduce the concentration of PM2.5 and PM10 by 20-30 per cent by 2030 taking 2000 as the base year.

Which of the statements given above is/are correct? a) 1 only b) 2 only c) Both 1 and 2 d) Neither 1 nor 2

Answer: d

Explanation:

Recently, Union Environment Minister launched the National Clean Air Programme (NCAP) in New Delhi.

Statement (1) is incorrect: NCAP is a time bound national level strategy for pan India implementation to tackle the increasing air pollution problem across the country in a comprehensive manner.

Statement (2) is incorrect: NCAP is a mid-term, five-year action plan with 2019 as the first year. The government has set a tentative national level target of 20%–30% reduction of PM 2.5 and PM 10 concentration by 2024 taking 2017 as the base year for the comparison of concentration.

102 non-attainment cities have been identified for implementing mitigation actions under NCAP. Institutional Framework at Centre and State Level comprising of Apex Committee at the Ministry of Environment Forest and Climate Change in the Centre and at Chief Secretary Level in the States are to be constituted.

Source: http://vajiramias.com/current-affairs/clean-air-plan/5c5131ba1d5def7a8516d020/

Q29. Consider the following statements:

(1) Swadesh darshan scheme is a national mission on pilgrimage rejuvenation and spiritual augmentation drive.

(2) Prasad scheme is an integrated development of theme-based tourist circuits in the country in 2014-15.

Which of the statements given above is/are correct? a) 1 only b) 2 only c) Both 1 and 2 d) Neither 1 nor 2

Answer: d

Explanation:

Statement (1) is incorrect: PRASAD Scheme is a national mission on pilgrimage rejuvenation and spiritual augmentation drive. Its objective is “development and beautification of the identified pilgrimage destinations”.

Statement (2) is incorrect: Swadesh darshan Scheme is Integrated Development of Theme- Based Tourist Circuits in a State and Union Territory.

Under these schemes, Ministry of Tourism provides Central Financial Assistance (CFA) to State/UT’s for various tourism projects. The Ministry of Tourism has sanctioned four new projects for Rs. 190.46 Crores under the tourism infrastructure development schemes, Swadesh Darshan and PRASAD in the states of Meghalaya, Gujarat and Uttar Pradesh recently. ‘Development of Govardhan’ in District Mathura in Uttar Pradesh has been sanctioned under the PRASHAD scheme. ‘Development of Pilgrimage Amenities at Somnath – Phase II’ has been sanctioned under the PRASHAD scheme.

Source: http://vajiramias.com/current-affairs/swadesh-darshan-and- prasad/5c3d91682099370eb957704e/

Q30. Consider the following statements with reference to the Human Space Flight Centre (HSFC):

(1) Its mandate is to carry out all activities related to the Gaganyaan Project.

(2) It is headquartered at Thumba, Thiruvananthapuram.

Which of the statements given above is/are correct? a) 1 only b) 2 only c) Both 1 and 2 d) Neither 1 nor 2

Answer: a

Explanation:

Statement (1) is correct: Recently, the Indian Space Research Organisation (ISRO) announced that work on ‘Gaganyaan’, the project to send a manned mission to space by 2022, would start soon at the newly created Human Space Flight Centre (HSFC). Its mandate is to carry out all activities related to the Gaganyaan Project.

Statement (2) is incorrect: It is headquartered at Bengaluru and Unnikrishnan Nair has been appointed as the director of the new centre.

Source: http://vajiramias.com/current-affairs/human-space-flight-centre- hsfc/5c5286441d5def7a8625fe91/

Q31. Which of the following persons are members of the committee to appoint the director of CBI?

(1) Chief Justice of India

(2) Prime Minister

(3) Leader of Opposition in the Lok Sabha

(4) Central Vigilance Commissioner

Select the correct answer using the code given below. a) 1 and 2 only b) 2, 3 and 4 only c) 1, 3 and 4 only d) 1, 2 and 3 only

Answer: d

Explanation:

The Central Bureau of Investigation (CBI) is the premier investigating agency of India. It investigates several economic crimes and special crimes. It is headquartered in New Delhi. It operates under the jurisdiction of the Ministry of Personnel, Public Grievances and Pensions.

The Central Government shall appoint the Director of CBI on the recommendation of a three- member committee consisting of the Prime Minister as Chairperson, the Leader of Opposition in the Lok Sabha and the Chief Justice of India or Judge of the Supreme Court nominated by him. Hence, option d) is the correct answer.

Source: http://vajiramias.com/current-affairs/central-bureau-of-investigation- cbi/5c3537542099376f20daa8e9/

Q32. Recently, six new species of bent-toed geckos were discovered from which of the following regions of India?

a) Western Ghats b) North Eastern India c) Eastern Ghats d) Andaman Islands

Answer: b

Explanation:

Geckos are reptiles and are found on all the continents except Antarctica. These colorful lizards have adapted to habitats from rain forests, to deserts, to cold mountain slopes. Recently, an international team of Researchers have published a study in Zootaxa, announcing the discovery of six new species of bent-toed geckos from north-eastern India. Bent-toed geckos are a diverse genus of a type of small lizard. This genus has more than 250 described species at present making it the largest of all gecko genera.

The six newly discovered species are Guwahati bent-toed gecko, Kaziranga bent-toed gecko, Jaintia bent-toed gecko, Nagaland bent-toed gecko, Abhayapuri bent-toed gecko (found near Abhayapuri town in Assam’s Bongaigaon district) and Jampui bent-toed gecko (found in Tripura’s Jampui Hills). Hence, option b) is the correct answer.

Source: http://vajiramias.com/current-affairs/bent-toed- geckos/5c3d952720993715d1e2b8e8/

Q33. Which of the following pairs is/are correctly matched?

Select the correct answer using the code given below: a) 1 only b) 1 and 2 only c) 2 and 3 only d) 1, 2 and 3

Answer: d

Explanation:

Pair (1) is correctly matched: Swami Vivekananda (1863 – 1902), born Narendranath Datta was an Indian Hindu monk and a chief disciple of the 19th century Indian mystic Ramakrishna. In 1897 he founded the Ramakrishna Math and the Ramakrishna Mission, a Indian socio-religious reform movement. He gave a new theory of ethics & morality based on the intrinsic purity and oneness of the Atman. In 1984 the Government of India declared that 12 January, the birthday of Swami Vivekananda, will be celebrated as National Youth Day. Vedanta Society of New York (VSNY) was the first Vedanta Society founded by Swami Vivekananda in New York in November 1894.

Pair (2) is correctly matched: Gazulu Lakshminarasu Chetty CSI (1806–1868) was an Indian merchant, Indian independence activist and political activist who founded the Madras Native Association in 1849 and the first Indian-owned newspaper in Madras, The Crescent.

Pair (3) is correctly matched: Poona Sarvajanik Sabha, was a socio-political organisation in British India which started with the aim of working as a mediating body between the government and people of India and to popularise the peasants' legal rights. It was started in 1870 by Mahadev Govind Ranade and his associates in Pune. Hence, option d) is the correct answer.

Source: http://vajiramias.com/current-affairs/swami- vivekananda/5c3d93c720993715cf9968bb/

Q34. With respect to “Guru Gobind Singh”, consider the following statements:

(1) He introduced the Five Ks, the five articles of faith that Khalsa Sikhs wear at all times.

(2) He composed Dasam Granth which includes compositions which are part of the daily prayers/lessons of Sikhs.

Which of the statements given above is/are correct?

a) 1 only b) 2 only c) Both 1 and 2 d) Neither 1 nor 2

Answer: c

Explanation:

Guru Gobind Singh (1666 – 1708), was the tenth Sikh Guru, warrior, poet and philosopher. He was born as Gobind Rai as the son of Guru Tegh Bahadur.

Statement (1) is correct: He founded the Sikh warrior community called Khalsa in 1699. He introduced the Five Ks (Kesh, Kangha, Kara, Kirpan and Kacchera), the five articles of faith that Khalsa Sikhs wear at all times.

Statement (2) is correct: He Finalized the Guru Granth Sahib in 1706 and declared this text to be the eternal Guru for Sikhs and also composed Dasam Granth which includes compositions which are part of the daily prayers/lessons of Sikhs.

Source: http://vajiramias.com/current-affairs/guru-gobind- singh/5c3d93fc20993715d1e2b8e4/

Q35. Which of the following are the major ports in India?

(1) Kakinada

(2) Deendayal

(3) Cochin

(4) Paradip

Select the correct answer using the code given below: a) 1 and 2 only b) 3 and 4 only c) 2, 3 and 4 only d) 1, 2, 3 and 4

Answer: c

Explanation:

Kakinada Port is located at Kakinada off the east coast of India. It is 170 km south of Visakhapatnam Port. It is a minor port.

The major ports in India have recorded a growth of 3.77% and together handled 518.6 Million Tonnes of cargo during the period April to December, 2018. Deendayal Port handled the highest volume of traffic. In 2017, The Ministry of Shipping renamed Kandla Port Trust as Deendayal Port after Pandit Deendayal Upadhyay. This was done by the Central Government, in exercise of powers conferred on it under Indian Ports Act, 1908. Kandla Port, located on the Gulf of Kutch in Gujarat, is one of the twelve major ports in the country.

Cochin Port or Kochi Port is a port on the Arabian Sea. An all-weather natural Port, and located strategically close to the busiest international sea routes Cochin is promoting a major liquid terminal, bulk terminal and maritime industries in its port based SEZs .

Paradip Port is a natural, deep-water port on the East coast of India in Jagatsinghpur district of Odisha. It is situated at confluence of the Mahanadi river and the Bay of Bengal. Hence, option c) is the correct answer.

Source: http://vajiramias.com/current-affairs/deendayal-port/5c3d92f62099370eba75f3ea/

Q36. Which of the following countries are located in the Balkan Peninsula?

(1) Republic of Macedonia

(2) Spain

(3) Serbia

(4) Norway

Select the correct answer using the code given below: a) 1 and 2 only b) 1 and 3 only c) 2, 3 and 4 only d) 1, 2, 3 and 4

Answer: b

Explanation:

The Balkans, also known as the Balkan Peninsula, is a geographic area in southeastern Europe. The Balkans are usually characterized as comprising Albania, Bosnia and Herzegovina, Bulgaria, Croatia, Kosovo, Republic of North Macedonia, Montenegro, Serbia, and Slovenia—with all or part of each of those countries located within the peninsula. Portions of Greece and Turkey are also located within the geographic region generally defined as the Balkan Peninsula.

The Republic of Macedonia is a country in the Balkan Peninsula in Southeast Europe. It is a landlocked country; it has borders with Kosovo to the northwest, Serbia to the north, Bulgaria to the east, Greece to the south, and Albania to the west. . In June 2018, Greece and Macedonia signed the “PRESPA AGREEMENT”, to rename the latter the Republic of North Macedonia. Recently, Parliamentarians of the Republic of Macedonia voted to change their country’s name to the “Republic of North Macedonia”. The move now awaits approval from Greece’s Parliament. Hence, option b) is the correct answer.

Source: http://vajiramias.com/current-affairs/republic-of-north- macedonia/5c3d95d420993715d1e2b8ea/

Q37. With reference to the National Clean Energy Fund (NCEF), consider the following statements:

(1) It is a fund created in 2010-11 for funding research and innovative projects in clean energy technologies of public sector or private sector entities.

(2) The Fund is managed by Indian Renewable Energy Development Agency.

Which of the statements given above is/are correct? a) 1 only b) 2 only c) Both 1 and 2 d) Neither 1 nor 2

Answer: a

Explanation:

Statement (1) is correct: The National Clean Energy Fund (NCEF) is a fund created in 2010-11. For funding research and innovative projects in clean energy technologies of public sector or private sector entities, upto the extent of 40% of the total project cost.

Statement (2) is incorrect: The Fund is designed as a non-lapsable fund under Public Accounts and with its secretariat in Department of Expenditure, Ministry of Finance. An Inter-Ministerial Group, chaired by the Finance Secretary in Ministry of Finance recommends projects eligible for funding under NCEF.

The 42 nd standing committee on energy in its recent report tabled in Parliament has recommended financial support to the stressed gas-based power projects in the country from National Clean Energy Fund (NCEF).

Source: http://vajiramias.com/current-affairs/national-clean-energy-fund- ncef/5c3d966820993715cf9968c6/

Q38. Consider the following statements with reference to the Technology Centre Systems Programme (TCSP):

(1) It was introduced by the Union Government in 1999 to facilitate new technology for making the Indian textile industry globally competitive and to reduce the capital cost for the textile industry.

(2) It is partially funded by the World Bank.

Which of the statements given above is/are correct? a) 1 only b) 2 only c) Both 1 and 2 d) Neither 1 nor 2

Answer: b

Explanation:

Statement (1) is incorrect: The Ministry of MSME announced that it will develop 20 more technology centres across the country in another 3-5 years under the Technology Centre Systems Programme (TCSP). Currently, 10 such centres are operational in different states of the country. The Programme is expected to improve the competitiveness of Micro, Small and Medium Enterprises (MSMEs) in key manufacturing industries across India by facilitating improved access to manufacturing technology, improving further the availability and employability of skilled workers and establishing strong focus in providing business & technical advisory services.

Statement (2) is correct: It is being funded with support in funds from the World Bank.

Source: http://vajiramias.com/current-affairs/technology-mission-centre-on-solar-energy- water-treatment/5c4aa8bc2099374346817e86/

Q39. Consider the following statements with reference to the Chakmas and Hajongs:

(1) The Hajongs are predominantly Buddhists, while Chakmas are Hindus.

(2) They were inhabitants of the northern Rakhine state in Myanmar who migrated to India.

Which of the statements given above is/are correct? a) 1 only b) 2 only

c) Both 1 and 2 d) Neither 1 nor 2

Answer: d

Explanation:

Recently, People in Arunachal Pradesh are protesting against the Citizenship (Amendment) Bill, 2016 as they argue that the bill would serve as a legal basis for legitimising the claims of Chakma and Hajong refugees as the indigenous people of their State.

Statement (1) is incorrect: The Chakmas and Hajongs are ethnic people. Chakmas are predominantly Buddhists; Hajongs are Hindus.

Statement (2) is incorrect: They were inhabitants of the Chittagong Hill Tracts of erstwhile East Pakistan (now Bangladesh) who migrated to India due to Submergence of their land by the Kaptai dam on the Karnaphuli River in the 1960s and religious persecution they faced in East Pakistan as they were non-Muslims.

Source: http://vajiramias.com/current-affairs/chakma-and- hajong/5c3dc1f220993715d1e2ba51/

Q40. With respect to “UNNATI programme”, recently seen in news, consider the following statements:

(1) It is a capacity building programme for India’s first human space mission, Gaganyaan.

(2) It has been launched by the Indian Space research Organisation (ISRO) as part of an initiative by the United Nations Office for Outer Space Affairs.

Which of the statements given above is/are correct? a) 1 only b) 2 only c) Both 1 and 2 d) Neither 1 nor 2

Answer: b

Explanation:

Statement (1) is incorrect: UNNATI is an acronym for Unispace Nanosatellite Assembly and Training programme. It is a capacity building programme on Nanosatellite development.

Statement (2) is correct: It is planned to be conducted for 3 years by U.R. Rao Satellite Centre of Indian Space research Organisation (ISRO) in 3 batches and will target to benefit officials of 45 countries. It has been launched by ISRO as part of an initiative by United Nations Office for Outer Space Affairs that is commemorating the 50th anniversary of the first United Nations Conference on the Exploration and Peaceful Uses of Outer Space (UNISPACE-50).

Source: http://vajiramias.com/current-affairs/unnati/5c416127209937072df28d89/

Q41. With respect to “Deendayal Disabled Rehabilitation Scheme (DDRS)”, consider the following statements:

(1) It is a scheme to provide financial assistance to NGOs working for education and rehabilitation of persons with disabilities.

(2) It is being implemented by the Ministry of Social Justice & Empowerment.

Which of the statements given above is/are correct? a) 1 only b) 2 only c) Both 1 and 2 d) Neither 1 nor 2

Answer: c

Explanation:

Statement (1) is correct: Deendayal Disabled Rehabilitation Scheme (DDRS) is a Central Sector Scheme of Government of India to provide financial assistance to NGOs working for education and rehabilitation of persons with disabilities.

Statement (2) is correct: It is being implemented by Ministry of Social Justice & Empowerment, Department of Empowerment of Persons with Disabilities.

Recently, a regional conference on “Deendayal Disabled Rehabilitation Scheme (DDRS)” was organized on 17th January, 2019 in Mumbai. This is the second of a series of Regional Conferences to be held across the country and the exercise will culminate in a National Conference in New Delhi.

Source: http://vajiramias.com/current-affairs/deendayal-disabled-rehabilitation-scheme- ddrs/5c41628c2099370703ecffa0/

Q42. Which one of the following is the best description of ‘INS Kohassa’, that was in the news recently? a) Indian naval air station b) Nuclear-powered submarine c) Torpedo launch and recovery vessel d) Nuclear-powered aircraft carrier

Answer: a

Explanation:

To enhance the operational capability of Andaman and Nicobar Command (ANC) Naval Air Station (NAS) has been commissioned as INS Kohassa on 24 January 2019 by Sunil Lanba, Chief of Naval Staff.

INS Kohassa has been named after a White- Bellied Sea Eagle, which is a large bird of prey endemic to Andaman and Nicobar Islands (ANI). It is located on the northern most inhabited island in North Andaman. INS Kohassa will become the third Naval Air Base in the Andaman and Nicobar islands after INS Utkrosh at and INS Baaz at Campbell Bay. NAS Shibpur was established in 2001 as a Forward Operating Air Base (FOAB) for surveillance in North Andaman. Hence, option a) is the correct answer.

Source: http://vajiramias.com/current-affairs/ins-kohassa/5c42b4382099370f26139ec8/

Q43. Consider the following statements with reference to the ‘proposed Kaveri Crater’:

(1) It located between Nilgiris and Kodaikanal.

(2) It is the Earth’s largest and only hyper-velocity impact crater in basaltic rock.

Which of the statements given above is/are correct? a) 1 only b) 2 only c) Both 1 and 2

d) Neither 1 nor 2

Answer: a

Explanation:

Statement (1) is correct: Kaveri Crater is between Nilgiris and Kodaikanal. Both the Palghat Gap and Dhimbam Ghats are a part of the Kaveri Crater, with the geological evidence stretching up to Belakavadi and Shivanasamudra in Karnataka. The area ‘Kaveri Crater’, is possibly the fourth largest in the world. The crater has a diameter of 120 km. Unlike smaller craters, Kaveri Crater can be visualised only through satellite images.

Statement (2) is incorrect: Lonar Lake is the Earth’s largest and only hyper-velocity impact crater in basaltic rock.

Findings of a Research study, funded by the Union Ministry of Earth Sciences, hints at asteroid strike 800 million to 550 million years ago, creating a crater in .. Their findings were published in the Journal of the Geological Society of India and the study won the Radhakrishna Prize 2018 for best paper.

Source: http://vajiramias.com/current-affairs/kaveri-crater/5c440b91209937269272c615/

Q44. Consider the following statements with reference to the World Capital of Architecture:

(1) Paris will be the first city to receive the title under this program.

(2) It is launched together by UNESCO and the International Union of Architects (UIA).

Which of the statements given above is/are correct? a) 1 only b) 2 only c) Both 1 and 2 d) Neither 1 nor 2

Answer: b

Explanation:

Statement (1) is incorrect: The United Nations Educational, Scientific, and Cultural Organization (UNESCO) has recently announced that the Brazilian city of Rio de Janeiro will be the World Capital of Architecture for 2020. Having defeated Paris and Melbourne, Rio will be the first city to receive the title under this program.

Statement (2) is correct: It is launched together by UNESCO and the International Union of Architects (UIA) in November last year.

The city will host the World Congress of UIA, in July 2020, an event that occurs every three years. According to UNESCO, the World Capital of Architecture is intended to become an international forum for debates about pressing global challenges from the perspectives of culture, cultural heritage, urban planning and architecture.

Source: http://vajiramias.com/current-affairs/world-capital-of- architecture/5c456504209937148a22b227/

Q45. Consider the following statements with reference to the total lunar eclipse:

(1) It takes place when the Earth comes between the Sun and the Moon and covers the Moon with its shadow.

(2) During totality, the moon looks red, also known as a blood moon, because of sunlight scattering off Earth’s atmosphere.

(3) In July, the full moon is also sometimes known as the wolf moon or Great Spirit moon.

Which of the statements given above are correct? a) 1 and 2 only b) 2 and 3 only c) 1 and 3 only d) 1, 2 and 3

Answer: a

Explanation:

Statement (1) is correct: On January 20 night/early January 21, depending on the location of the moon, Earth and sun lined up for the only total lunar eclipse this year and next. At the same time, Earth also witnessed a supermoon as the Moon will be closer to the planet and will appear slightly bigger and brighter than usual. Total lunar eclipse takes place when the Earth comes between the Sun and the Moon and covers the Moon with its shadow.

Statement (2) is correct: During totality, the moon looks red, also known as a blood moon, because of sunlight scattering off Earth’s atmosphere.

Statement (3) is incorrect: In January, the full moon is also sometimes known as the wolf moon or Great Spirit moon.

Source: http://vajiramias.com/current-affairs/total-lunar- eclipse/5c455d8f209937128a013873/

Q46. Consider the following statements with reference to the Saturn’s Rings:

(1) The rings consist of a large number of small particles that are either icy snowballs or ice covered rocks.

(2) Pioneer 11 was the first mission to explore Saturn.

Which of the statements given above is/are correct? a) 1 only b) 2 only c) Both 1 and 2 d) Neither 1 nor 2

Answer: c

Explanation:

A recent study published in Science suggests that the Saturn’s rings formed between 100 million and 10 million years ago, and are thus billions of years younger than the 4.5- billion- year-old Saturn. Although the rings were first observed by telescopes centuries ago, knowledge about their formation and composition has been relatively slow coming from four robotic spacecraft that have visited Saturn — Pioneer 11, Voyager 1, Voyager 2 and Cassini.

Statement (1) is correct: The rings consist of a large number of small particles that are either icy snowballs or ice covered rocks.

Statement (2) is correct: Pioneer 11 was the first mission to explore Saturn and the second spacecraft to fly past and study Jupiter.

Source: http://vajiramias.com/current-affairs/saturns-rings/5c455eb8209937128a0138db/

Q47. The Kerch Strait, recently seen in news, is connecting which of the following water bodies? a) Adriatic Sea and Ionian Sea b) Gulf of Mexico and Caribbean Sea c) Black Sea and Sea of Azov d) Pacific and South Atlantic Sea

Answer: c

Explanation:

The Kerch Strait is a strait connecting the Black Sea and the Sea of Azov. It separates the Kerch Peninsula of Crimea from the Taman Peninsula of Russia’s Krasnodar Krai. Recently, two ships carrying Indian, Turkish and Libyan crew members caught fire in the Kerch Strait, killing at least 14 sailors. The fire broke out as the two ships were transferring fuel from one to the other.

Source: http://vajiramias.com/current-affairs/kerch-strait/5c47fd4a2099372c743f6691/

Q48. Consider the following statements with reference to the Cinereous vulture:

(1) It is a large raptorial bird that is distributed through much of Africa.

(2) It is listed as Vulnerable on IUCN Red list of threatened species.

Which of the statements given above is/are correct? a) 1 only b) 2 only c) Both 1 and 2 d) Neither 1 nor 2

Answer: d

Explanation:

Cinereous vulture was recently spotted in Hazaribagh in Jharkhand, with Birdwatchers calling it a rare sighting.

Statement (1) is incorrect: The cinereous vulture is one of the heaviest and largest raptors in the world distributed through much of Eurasia. It is a dark brown and broad-winged species with a slightly wedge-shaped tail. The bald head and neck are a bluish gray, with a fluffy collar which is lighter in older birds. In many countries, this bird is called ‘monk vulture’, because of its upright standing neck feathers that resemble the hood of a monk.

Statement (2) is incorrect: It is listed as Near Threatened on IUCN Red list of threatened species.

Source: http://vajiramias.com/current-affairs/cinereous- vulture/5c3529d92099376f2119ed15/

Q49. National Voters’ Day is celebrated on January 25 to mark: a) The Foundation day of Election Commission of India. b) First election to the Lok Sabha since India became independent in August 1947. c) Decision to hold elections on the basis of Universal Adult Suffrage in India. d) Enactment of Sixty-first Amendment of the Constitution of India which lowered the voting age of elections from 21 years to 18.

Answer: a

Explanation:

The 9th National Voters’ Day has been celebrated across the country on January 25 for enhanced participation of citizens in the electoral process. It is celebrated on January 25 to mark the Foundation day of Election Commission of India, which was established on 25th January 1950. This years’ theme is “No Voter to be Left Behind”. The National Awards for the Best Electoral Practices will be conferred on officers for their outstanding performance in the conduct of elections. Hence, option a) is the correct answer.

Source: http://vajiramias.com/current-affairs/national-voters- day/5c4aac522099374346817ef7/

Q50. Consider the following statements with reference to the South Asian Nitrogen Hub (SANH):

(1) It is an initiative under Conference of the Parties to the United Nations Framework Convention on Climate Change (COP24), Katowice to assess and study the quantum and impact of “nitrogen pollution” in South Asia.

(2) It will look at nitrogen in agriculture in eight countries — India, Pakistan, Bangladesh, Nepal, Afghanistan, Sri Lanka, Bhutan and Maldives.

Which of the statements given above is/are correct? a) 1 only b) 2 only c) Both 1 and 2 d) Neither 1 nor 2

Answer: b

Explanation:

Statement (1) is incorrect: Eighteen research institutions in India are among a group of 50 institutions called the South Asian Nitrogen Hub (SANH) in the United Kingdom and South Asia. The group has secured £20 million (about ₹200 crore) from the U.K. government to assess and study the quantum and impact of “nitrogen pollution” in South Asia.

Statement (2) is correct: The SANH will study the impacts of the different forms of pollution to form a “coherent picture” of the nitrogen cycle. In particular, it will look at nitrogen in agriculture in eight countries — India, Pakistan, Bangladesh, Nepal, Afghanistan, Sri Lanka, Bhutan and Maldives.

Source: http://vajiramias.com/current-affairs/south-asian-nitrogen-hub- sanh/5c4aa9642099374347cf6c28/

Q51. Right to Information Act, 2005 is regarded as one of the finest legislations in the world in bringing accountability in governance. In this context which of the following as “information” comes under the Right to Information Act?

(1) Electronic Voting Machine (EVM)

(2) Unaided religious trusts

(3) Supreme Court

(4) Jammu and Kashmir Bank

Select the correct answer using the code given below: a) 1 and 4 only b) 2 and 3 only c) 2 and 4 only d) 1, 2 and 3 only

Answer: a

Explanation:

Point (1) is correct: The Central Information Commission (CIC) has ruled that an Electronic Voting Machine (EVM) is “information” under the Right to Information Act. The CIC ruled that “the EVM which is available with the ECI in a material form and also as samples is an information under the RTI Act.” The definition of information under Section 2(f) of the RTI Act includes “any material in any form, including records, documents, memos, e-mails, opinions, advices, press releases, circulars, orders, logbooks, contracts, reports, papers, samples, models, data material held in any electronic form…”

Point (2) is incorrect: Recently, Hyderabad high court has held that religious institutions like temples, churches and mosques that are not funded by the government do not fall within the purview of the Right to Information Act-2005 (RTI).

Point (3) is incorrect: The Supreme Court recently lauded the role of the Right to Information (RTI) Act as an “integral part of any vibrant democracy.” But the apex court itself has refused to come under the ambit of the information transparency law for the past one decade.

Point (4) is correct: Recently the Jammu and Kashmir bank has been brought under the purview of the Right to Information (RTI) act. The State Administrative Council (SAC) approved the proposal for treating the J&K Bank Limited as a Public Sector Undertaking (PSU).

Source: http://vajiramias.com/current-affairs/evm-is-information-under- rti/5c73a8301d5def712bba6b85/ http://vajiramias.com/current-affairs/jk-bank/5c1e076420993740479d2855/

Q52. Which of the following best describes the term ‘Gynandromorphs’, recently seen in news? a) Organisms showing both female and male characteristics. b) A recently discovered exoplanet that orbits Barnard's Star. c) A ransomware worm that spread rapidly through across a number of computer networks. d) Condition where the two sexes of the same species exhibit different characteristics beyond the differences in their sexual organs.

Answer: a

Explanation:

According to researchers, split-Sex animals are unusual but not as rare as we believed. From butterflies to chickens to lobsters, mixed male-female bodies offer clues as to why certain

diseases strike one gender more often than the other. Gynander or Gynandromorphs are the organisms showing both female and male characteristics. The term is derived from the Greek words (gyne = woman; aner = man and morphe = form). These organisms are found in variety of species ranging from butterflies to chickens to lobsters but are most notable in butterflies, moths and other insects. Hence, option a) is the correct answer.

Source: http://vajiramias.com/current- affairs/gynandromorphs/5c76615a1d5def14e2964e29/

Q53. Which of the following bodies/institutions are headed by the Prime Minister of India?

(1) National Board for Wildlife

(2) National Integration Council

(3) National Ganga Council

(4) Zonal Councils

Select the correct answer using the code given below: a) 1 and 4 only b) 2 and 3 only c) 1, 2 and 3 only d) 1, 2, 3 and 4

Answer: c

Explanation:

Point (1) is correct: National Board for Wildlife (NBWL) is a statutory body as it has been constituted under under Section 5A the Wildlife Protection Act, 1972. It is the apex body to review all wildlife-related matters and approve projects in and around national parks and sanctuaries i.e. Protected Areas. It is a 47-member board (including the chairman) which usually meets once a year. It is chaired by Prime minister.

Point (2) is correct: The National Integration Council (NIC) was constituted in 1961, following a decision taken at a national conference on ‘unity in diversity’, convened by the Central government, at New Delhi. It consisted of the prime minister as chairman, central home minister, chief ministers of states, seven leaders of political parties, the chairman of the UGC,

two educationists, the commissioner for SCs and STs and seven other persons nominated by the prime minister. The last meeting of which was held in September 2013.

Point (3) is correct: National Mission for Clean Ganga (NMCG) was registered as a society on 12th August 2011 under the Societies Registration Act 1860.It acted as implementation arm of National Ganga River Basin Authority (NGRBA) which was constituted under the provisions of the Environment (Protection) Act (EPA), 1986. NGRBA has since been dissolved with effect from the 7th October 2016, consequent to constitution of National Council for Rejuvenation, Protection and Management of River Ganga (referred as National Ganga Council). National Ganga Council under chairmanship of Hon’ble Prime Minister of India. The Empowered Task Force (ETF) on river Ganga under chairmanship of Hon’ble Union Minister of Water Resources, River Development and Ganga Rejuvenation.

Point (4) is not correct: The Zonal Councils are the statutory (and not the constitutional) bodies. They are established by an Act of the Parliament, that is, States Reorganisation Act of 1956. The act divided the country into five zones (Northern, Central, Eastern, Western and Southern) and provided a zonal council for each zone. Each zonal council consists of the following members: a) home minister of Central government. b) chief ministers of all the States in the zone. c) Two other ministers from each state in the zone. d) Administrator of each union territory in the zone. Besides, the following persons can be associated with the zonal council as advisors (i.e., without the right to vote in the meetings): (i) a person nominated by the Planning Commission; (ii) chief secretary of the government of each state in the zone; and (iii) development commissioner of each state in the zone. The home minister of Central government is the common chairman of the five zonal councils. Each chief minister acts as a vice-chairman of the council by rotation, holding office for a period of one year at a time.

Source: http://vajiramias.com/current-affairs/national-board-of-wildlife- nbwl/5c651f7a1d5def2a98342ad7/

Q54. With reference to the Earth’s magnetism, consider the following statements:

(1) The origin of Earth’s magnetism lies in its outer core.

(2) The magnetic north pole or South Pole coincides with the geographical north or South Pole.

(3) The North magnetic pole isn’t static but moving.

Which of the statements given above are correct? a) 1 and 2 only b) 2 and 3 only

c) 1 and 3 only d) 1, 2 and 3

Answer: c

Explanation:

Statement (1) is correct: The origin of Earth’s magnetism lies in its outer core, a more than 2,000-km layer of liquid iron and some other metals like nickel that surrounds the central core, or the innermost part. This liquid iron is in constant motion due to Earth’s rotation and various other reasons, and this motion produces a magnetic field.

Statement (2) is incorrect: The magnetic north pole, or South Pole, does not coincide with the geographical north or South Pole.

Statement (3) is correct: Currently, the magnetic north pole is located somewhere over northern Canada, a fact discovered in 1831 by Sir James Clark Ross. The north magnetic pole is moving towards Siberia. This movement has suddenly increased, quite significantly, from about 14-15 km per year till the 1990s to about 55 km per year in the last few years. This led to scientists updating the World Magnetic Model (WMM) that tracks this movement.

Source: http://vajiramias.com/article/magnetic-north-pole/5c5a9a7d1d5def7a89822482/

Q55. Recently, the Union Minister of state for health has launched the National Action Plan - Viral Hepatitis at Mumbai to combat hepatitis and achieve countrywide elimination of which among the following Hepatitis by 2030? a) Hepatitis A b) Hepatitis E c) Hepatitis D d) Hepatitis C

Answer: d

Explanation:

Recently the Union minister of state for health has launched the National Action Plan - Viral Hepatitis at Mumbai to combat hepatitis and achieve countrywide elimination of Hepatitis C by 2030. The programme is in line with the country's global commitment towards achieving SDG 3.3. Hepatitis A and E are typically caused by ingestion of contaminated food or water (transmitted via oral-faecal route).Hepatitis B, C and D are transmitted through unsafe blood transfusions or contaminated needles/syringes (particularly among the drug users), sexual- transmission or even mother-to-child transmission. There is no vaccine for Hepatitis C. Hence, option d) is the correct answer.

Source: http://vajiramias.com/current-affairs/national-action-plan-viral- hepatitis/5c73a8c61d5def712e4e30c3/

Q56. With respect to the “Pradhan Mantri Kisan Samman Nidhi (PM-KISAN)”, consider the following statements:

(1) Under this programme, vulnerable landholding farmer families, having cultivable land upto 4 hectares, will be provided direct income support.

(2) Income support will be transferred directly into the bank accounts of beneficiary farmers, in three equal installments of Rs 2,000 each.

Which of the statements given above is/are correct? a) 1 only b) 2 only c) Both 1 and 2 d) Neither 1 nor 2

Answer: b

Explanation:

Statement (1) is incorrect: Pradhan Mantri Kisan Samman Nidhi (PM-KISAN) scheme’s objective is to provide an assured income support to small and marginal farmers. Under this programme, vulnerable landholding farmer families, having cultivable land upto 2 hectares, will be provided direct income support at the rate of Rs 6,000 per year.

Statement (2) is correct: This income support will be transferred directly into the bank accounts of beneficiary farmers, in three equal installments of Rs 2,000 each. This programme will be funded by Government of India and will entail an annual expenditure of Rs 75,000 crore.

Source: http://vajiramias.com/current-affairs/pradhan-mantri-kisan-samman-nidhi-pm- kissan/5c5546171d5def7a8981b6c9/ http://vajiramias.com/current-affairs/pm-kisan/5c73a6b81d5def712bba6b4c/

Q57. Which of the following statements is not correct about the polar vortex? a) The polar vortex is a large area of high pressure and cold air surrounding the Earth’s North and South poles.

b) The term vortex refers to the counter-clockwise flow of air that helps keep the colder air close to the poles. c) It spins in the stratosphere, a layer of the atmosphere 10-48 km above the ground and above the troposphere. d) North America and even parts of Europe and Asia had experienced cold surges connected to the polar vortex.

Answer: a

Explanation:

A record-breaking cold wave has swept through the United States caused by a blast of Arctic air, which in turn is a result of “Polar Vortex” event. The polar vortex is a large area of low pressure and cold air surrounding the Earth’s North and South poles.

The term vortex refers to the counter-clockwise flow of air that helps keep the colder air close to the poles. Often during winter in the Northern Hemisphere, the polar vortex will become less stable and expand, sending cold Arctic air southward over the United States with the jet stream. This is called a polar vortex event (“breaking off” of a part of the vortex). Due to this, North America, and even Portions of Europe and Asia also experience cold surges connected to the polar vortex.The polar vortex spins in the stratosphere, a layer of the atmosphere 10-48 km above the ground and above the troposphere, where most familiar weather patterns develop. Hence, option a) is the correct answer.

Source: http://vajiramias.com/current-affairs/polar-vortex/5c53e0821d5def7a89819884/

Q58. India Infrastructure Finance Company Limited (IIFCL) is a wholly-owned Government of India company set up in 2006 to provide long term finance to viable infrastructure projects. Which of the following sectors are eligible for financial assistance from IIFCL?

(1) Agriculture

(2) Cold storage chains

(3) International convention centres

(4) Fertilizer Manufacturing Industry

Select the correct answer using the code given below: a) 1 and 3 only b) 2 and 4 only c) 2, 3 and 4 only d) 1, 2, 3 and 4 only

Answer: c

Explanation:

The government plans to channelise India Infrastructure Finance Company Limited (IIFCL) for financing social sector infrastructure projects. Apart from funding mainstream infrastructure projects, which the state-owned IIFCL presently does, the company is being tasked to invest aggressively in educational and health infrastructure as well. It was set up in 2006 to primarily bridge funding deficit for infrastructure sector, is now being geared to support social sector infra projects, especially in the health and education sector.

The sectors eligible for financial assistance from IIFCL are as per the Harmonized list of Infrastructure Sub-Sectors as approved by the Government and RBI and as amended from time to time. These broadly include transportation, energy, water, sanitation, communication, social and commercial infrastructure. Only projects pertaining to following sectors are eligible for financing from IIFCL: Road and bridges, railways, seaports, airports, inland waterways and other transportation projects; Power; Urban transport, water supply, sewage, solid waste management and other physical infrastructure in urban areas; Gas pipelines; Infrastructure projects in Special Economic Zones; International convention centres and other tourism infrastructure projects; Cold storage chains; Warehouse and Fertilizer Manufacturing Industry.

Source: http://vajiramias.com/current-affairs/india-infrastructure-finance-company-limited- iifcl/5c724b001d5def712e4e0928/

Q59. With respect to the “Organisation of Islamic Cooperation (OIC)” consider the following statements:

(1) It is the second largest inter-governmental organization after the United Nations.

(2) India, with the third largest Muslim population in world, has an observer status with OIC.

Which of the statements given above is/are correct? a) 1 only b) 2 only c) Both 1 and 2 d) Neither 1 nor 2

Answer: a

Explanation:

Statement (1) is correct: Organisation of Islamic Cooperation (OIC) is an international organization based in Jeddah, Saudi Arabia with the primary objective of safeguarding the interests of the Muslim world in the spirit of promoting international peace and harmony. It is the second largest inter-governmental organization after the United Nations with a membership of 57 states. Of these, 47 are Muslim Majority countries.

Statement (2) is incorrect: India had shown an interest in joining the OIC as a member state at the time of its formation. However, India has never since made a formal application to join OIC as an observer or as a member state.

Source: http://vajiramias.com/current-affairs/organisation-of-islamic-cooperation- oic/5c720f531d5def712a3581d7/

Q60. Consider the following statements with reference to the Earthquake:

(1) The location below the earth’s surface where the earthquake starts is called the epicenter.

(2) Numerous earthquakes which occur locally over an extended period without a clear sequence of foreshocks, main quakes and aftershocks is known as Earthquake swarms.

Which of the statements given above is/are correct? a) 1 only b) 2 only c) Both 1 and 2 d) Neither 1 nor 2

Answer: b

Explanation:

Dahanu town in Maharashtra’s Palghar district has been hit by some 30 low-intensity earthquakes since November last year. Data collected so far point to an “earthquake swarm”.

Statement (1) is incorrect: An earthquake happens when two blocks of the earth suddenly slip past one another and the location below the earth’s surface where the earthquake starts is called the hypocenter and the location directly above it on the surface of the earth is called the epicenter.

Statement (2) is correct: Numerous earthquakes which occur locally over an extended period without a clear sequence of foreshocks, main quakes and aftershocks is known as Earthquake swarms.

Bureau of Indian Standards has grouped the country into four seismic zones, viz. Zone II, III, IV and V. Of these, Zone V is seismically the most active region, while zone II is the least. Zone - IV covers parts of Gujarat and small portions of Maharashtra near the west coast and Rajasthan etc. Zone – III comprises Kerala, Goa, Lakshadweep islands, and remaining parts of Maharashtra, Tamilnadu and Karnataka etc.

Source: http://vajiramias.com/current-affairs/why-is-palghar-being-repeatedly-jolted-by- earthquakes/5c59424c1d5def7a86268cfc/

Q61. Consider the following statements with reference to the Financial Action Task Force (FATF):

(1) It is an inter-governmental body established in 1989 as a global standard-setting body for anti-money laundering and combating the financing of terrorism (AML/CFT).

(2) FATF also comprises two regional organisations - the Gulf Cooperation Council and the European Commission.

Which of the statements given above is/are correct? a) 1 only b) 2 only c) Both 1 and 2 d) Neither 1 nor 2

Answer: c

Explanation:

Statement (1) is correct: The Financial Action Task Force (FATF) is an inter-governmental body established in 1989 as a global standard-setting body for anti-money laundering and combating the financing of terrorism (AML/CFT). It mandate is to set standards and promote effective implementation of legal, regulatory and operational measures for combating money laundering, terrorist financing and other related threats to the integrity of the international financial system.

Statement (2) is correct: The FATF currently comprises 36 member jurisdictions and 2 regional organisations (the Gulf Cooperation Council or GCC, and the European Commission). While India, Russia and China are members, Pakistan is only an associate member of Asia Pacific Group-FATF.

Recently, FATF has decided not to remove Pakistan from the ‘grey list’ for its failure to stop funding of terror groups, as Pakistan had lobbied for, but also did not accept an Indian demand to move Pakistan to the ‘black list’ yet.

Source: http://vajiramias.com/current-affairs/fatf-grey-list/5c71113e1d5def6643a8c695/

Q62. The Bombay Natural History Society (BNHS) in collaboration with NGOs and local birdwatchers across the country have taken up the first-ever count of flamingos in India. Which among the following species of flamingos are found in India?

(1) Greater Flamingo

(2) Lesser Flamingo

(3) James's flamingo

Select the correct answer using the code given below: a) 1 and 3 only b) 1 and 2 only c) 2 and 3 only d) 1, 2 and 3

Answer: b

Explanation:

‘Flamingo Count’ will be conducted in two phases at a pan-India level. In India there are two species of flamingos

Point (1) is correct: Greater Flamingo (Phoenicopterus rosues): They are widespread in India, and they migrate to South India during winter and spend their time in large reservoirs and mud flats. IUCN status: Least concern.

Point (2) is correct: Lesser Flamingo (Phoeniconaias minor): They mainly breed at the Rann of Kutch/North- western India. IUCN status: Not threatened.

Point (3) is incorrect: James's flamingo, also known as the puna flamingo, is a species of flamingo that populates the high altitudes of Andean plateaus of Peru, Chile, Bolivia, and Argentina. It is named for Harry Berkeley James, a British naturalist who studied the bird.

Source: http://vajiramias.com/current-affairs/flamingo-count/5c7106ec1d5def6644a6f1be/

Q63. With respect to the “HACHIMOJI DNA”, recently seen in news, consider the following statements:

(1) It is a new DNA-like molecular system that can store and transmit information.

(2) Unlike DNA which has four nucleotides, the new synthetic DNA molecule has eight nucleotides.

Which of the statements given above is/are correct?

a) 1 only b) 2 only c) Both 1 and 2 d) Neither 1 nor 2

Answer: c

Explanation:

Statement (1) is correct: Researchers from the University of Texas and Indiana University in the US have synthesised Hachimoji DNA, a DNA-like molecular system to aid search for alien life. It is a synthesised molecular system that like DNA can store and transmit information.

Statement (2) is correct: The new informational molecular system is like DNA, except in one key area. DNA components include four key ingredients called nucleotides. However, the new synthetic DNA molecule has eight informational ingredients instead of four. It includes the four nucleotides present in Earth life -- adenine, cytosine, guanine, and thymine -- but also four others that mimic the structures of the informational ingredients in regular DNA. This new molecular system, which is not a new life form, suggests scientists looking for life beyond Earth may need to rethink what they are looking for.

Source: http://vajiramias.com/current-affairs/hachimoji-dna/5c710e6a1d5def6641608427/

Q64. In the context of which of the following do you sometimes find the terms ‘Kundan’, ‘Meenakari’, ‘Bidri’, ‘filigree’, and Jadau’, in the news? a) Textiles b) Traditional Jewellery c) Rajput Paintings d) Ritual theatre of the Garhwal Himalayas

Answer: b

Explanation:

Union Minister of Commerce & Industry dedicated to the Nation 1000 crore worth of projects of his Ministry through video conference in New Delhi recently. The projects were inaugurated in 7 States and 2 UTs across the country. He laid the foundation stone on Common Facility Centre (CFC) in Coimbatore, Tamil Nadu. It has the capacity to train 50, 000 people in the

unique jewellery manufacture like Kundan, Meenakari, Bidri, temple jewellery, filigree and Jadau jewellery.

Kundan is jewelry that is embedded with glass beads. Polki uses the same technique, the only difference being that the stones here are not glass but uncut diamonds called Polki. Meenakari is the art of using enamelling in jewelry. And finally, Jadau is a technique that also uses Polki, Meenakari and Kundan but the technique varies a bit from Kundan work. Jad means to be embedded and that is the technique used by artisans who craft this style of Jewelry. As with many others of our jewelry traditions, Jadau was brought into the country by the Mughals. It was then perfected by the Indian craftsmen of Rajasthan, specifically Bikaner. Today Rajasthan and Gujarat are the most well-known states for Jadau work. Hence, option b) is the correct answer.

Source: http://vajiramias.com/current-affairs/new-commerce-industry-ministry-projects- inaugurated/5c710d511d5def6643a8c629/

Q65. Consider the following statements with reference to the ‘Yuva Sahakar’ Cooperative Enterprise Support and Innovation Scheme:

(1) Its objective is to encourage cooperative start-ups and innovative projects in agriculture and allied sectors by providing them cheaper loans.

(2) Small Industries Development Bank of India (SIDBI) is the implementing agency of this scheme.

Which of the statements given above is/are correct? a) 1 only b) 2 only c) Both 1 and 2 d) Neither 1 nor 2

Answer: a

Explanation:

Statement (1) is correct: ‘Yuva Sahakar’ scheme’s objective is to encourage cooperative start- ups and innovative projects in agriculture and allied sectors by providing them cheaper loans.

Statement (2) is incorrect: National Cooperative Development Corporation (NCDC) is the implementing agency. A corpus fund of Rs 1,000 crore has been created by NCDC for this purpose. The scheme has component of subsidy as well as interest subvention.

Source: http://vajiramias.com/current-affairs/yuva-sahakar-cooperative-enterprise-support- and-innovation-scheme/5c77c3931d5def14e2967c94/

Q66. Which of the following temple is regarded as the ‘Women’s Sabarimala’? a) Attukal Bhagavathy Temple, Kerala b) Madurai Meenakshi Temple, Tamil Nadu c) Sri Amirthagateswarar Abhirami Temple, Tamil Nadu d) Chottanikkara Bhagavathy Temple, Kerala

Answer: a

Explanation:

Attukal Bhagavathy Amman Temple in Thiruvananthapuram, Kerala is famous for the annual Pongala festival. Only women participate in this annual Pongala festival by cooking Pongala (a mix of rice, jaggery and scraped coconut) and offering it as the prasad to the presiding deity. The event was listed in the Guinness World Records for being the largest religious gathering of women on a single day.

Attukal Bhagavathy temple, Kerala is also famously known as 'women's Sabarimala' as only women perform the rituals. Hence, option a) is the correct answer.

Source: http://vajiramias.com/current-affairs/attukal-pongala/5c6e76671d5def200edee103/

Q67. Which of the following best describes the term “Mariculture”, recently seen in news? a) It is the farming of aquatic plants and animals in salt water, either in the natural marine environment, or in land- or sea-based enclosures. b) It is the farming of aquatic animals and plants using inland sources of saline ground water. c) It provides the byproducts, including waste, from one aquatic species as inputs for another. d) It is an aquaculture business designed to raise and produce freshwater prawns or shrimps for human consumption.

Answer: a

Explanation:

Mariculture is the farming of aquatic plants and animals in salt water, either in the natural marine environment, or in land- or sea-based enclosures, such as cages, ponds, or raceways. Thus, mariculture represents a subset of the larger field of aquaculture, which involves the farming of both fresh-water and marine organisms. The major categories of mariculture species are: seaweeds, mollusks, crustaceans, and finfish.

Recently, the experts at Central Salt and Marine Chemicals Research Institute (CSMCRI) at Bhavnagar, Gujarat have suggested India should pursue Mariculture for ensuring food security, given its 7,500 km-long coastal line. The government of India has decided to promote 'marine culture fisheries' and included the sub-components of ‘Mariculture' under 'Blue Revolution' Scheme. The Department of Animal Husbandry, Dairying & Fisheries (DADF) has formulated a document on Mission Mariculture-2022 to promote mariculture including open sea cage culture activity in all maritime States and UTs on priority basis.

Source: http://vajiramias.com/current-affairs/mariculture/5c691ea81d5def3d3e0e7b8e/

Q68. Which of the following pairs is/are correctly matched?

Select the correct answer using the code given below: a) 1 only b) 1 and 2 only c) 2 and 3 only d) 1, 2 and 3

Answer: a

Explanation:

Pair (1) is correctly matched: The Thane Creek Flamingo Sanctuary (TCFS) is on the western bank of the creek, between the Airoli and the Vashi bridges connecting Mumbai and Navi Mumbai. It came into being in 2015 and is Maharashtra’s second marine sanctuary, after Malvan.

Pair (2) is not correctly matched: Betla National Park is a national park located on the Chhota Nagpur Plateau in the Latehar district of Jharkhand, India. The park hosts a wide variety of wildlife. The core area of Palamau Forest, the Betla National Park, is the main attraction in the forest.

Pair (3) is not correctly matched: The Fakim Sanctuary lies in the Kiphire district in Nagaland. It is close to the Myanmar border. The sanctuary is considered to be an abode for many wildlife animals such as leopards, tigers, wild buffaloes, hoolock gibbons and mithun. Hornbill, the most popular bird in Nagaland is also abundantly found in the Fakim Wildlife Sanctuary. It was established in the year 1983.

Source: http://vajiramias.com/current-affairs/thane-creek-flamingo-sanctuary- tcfs/5c6918cb1d5def3d40d0e1db/ http://vajiramias.com/current-affairs/north-koel-mandal-dam- project/5c3535522099376f2119ed3c/ http://vajiramias.com/current-affairs/fakim-wildlife-sanctuary/5c9f1cff1d5def0854ef69da/

Q69. Consider the following statements with reference to the Most Favoured Nation (MFN) status:

(1) Article 1 of General Agreement on Tariffs and Trade (GATT), 1994, requires every WTO member country to accord MFN status or preferential trade terms with respect to tariffs and trade barriers to all other member countries.

(2) India accorded MFN status to all WTO member countries, including Pakistan, from the date of entry into force of the so called Marrakesh Agreement, establishing the WTO.

Which of the statements given above is/are correct? a) 1 only b) 2 only c) Both 1 and 2

d) Neither 1 nor 2

Answer: c

Explanation:

India has withdrawn the Most Favoured Nation (MFN) status granted to Pakistan following the Pulwama terror attack in Jammu and Kashmir.

Statement (1) is correct: Article 1 of General Agreement on Tariffs and Trade (GATT), 1994, requires every WTO member country to accord MFN status (or preferential trade terms with respect to tariffs and trade barriers) to all other member countries.

Statement (2) is correct: Accordingly, India accorded MFN status to all WTO member countries, including Pakistan, from the date of entry into force of the so called Marrakesh Agreement, establishing the WTO.

The WTO is the only global international organisation dealing with the rules of trade between nations and the 164 member countries of the WTO represent 98 per cent of world trade. In accordance with the MFN principle and its obligations under the WTO,

India accorded Pakistan MFN status in 1996. However, Pakistan is yet to transition fully to MFN status for India and it maintains a Negative List of 1,209 products that are not allowed to be imported from India.

Source: http://vajiramias.com/current-affairs/most-favoured- nation/5c67cb2c1d5def3d4298abd9/

Q70. With reference to the ‘NASA’S Opportunity rover mission’, consider the following statements:

(1) Opportunity was sent to Meridiani Planum, a plain just south of the Martian Equator.

(2) Opportunity was the first rover to identify and characterise sedimentary rocks on a planet other than Earth.

Which of the statements given above is/are correct? a) 1 only b) 2 only c) Both 1 and 2 d) Neither 1 nor 2

Answer: c

Explanation:

NASA has announced the end of the Opportunity rover’s mission. Opportunity rolled out on to the Martian surface in 2004, 20 days after its twin, Spirit, had landed on the other side of the Mars. NASA has also released a factsheet on the two rovers. NASA’s Mars Exploration Rover (MER) mission was a robotic space mission involving two Mars rovers, Spirit and Opportunity with the objective of exploring the Martian surface and geology.

Statement (1) is correct: Opportunity was sent to Meridiani Planum, a plain just south of the Martian Equator. Both Missions discovered that Mars was likely wetter and warmer in the past.

Statement (2) is correct: Opportunity was the first rover to identify and characterise sedimentary rocks on a planet other than Earth.

Source: http://vajiramias.com/current-affairs/nasas-opportunity- rover/5c6671911d5def3d42988bab/

Q71. Consider the following statements with reference to the ‘Special Provisions for Delhi’:

(1) The 69th Constitutional Amendment Act of 1991provided a special status to the Union Territory of Delhi.

(2) It created a legislative assembly and a council of ministers for Delhi and empowered the legislative assembly to make laws on all the matters of the State List and the Concurrent List.

(3) According to the recent Supreme Court verdict the elected Delhi government will have the right to appoint public prosecutors and inquiry commissions.

Which of the statements given above is/are correct? a) 1 only b) 2 and 3 only c) 1 and 3 only d) 1, 2 and 3

Answer: a

Explanation:

Statement (1) is correct: The 69th Constitutional Amendment Act of 1991 provided a special status to the Union Territory of Delhi, and redesignated it the National Capital Territory of Delhi and designated the administrator of Delhi as the lieutenant (lt.) governor.

Statement (2) is incorrect: It created a legislative assembly and a council of ministers for Delhi. The assembly can make laws on all the matters of the State List and the Concurrent List except the three matters of the State List, that is, public order, police and land. But, the laws of Parliament prevail over those made by the Assembly.

Statement (3) is incorrect: A two-judge bench of the Supreme Court recently gave its verdict on the six matters pertaining to a conflict between the Centre and the AAP government in Delhi. Both judges agreed that the Lieutenant Governor (L-G) will have control over the Anti- Corruption Bureau, as already notified by the Centre.The power to appoint inquiry commissions would also rest with the central government. The elected Delhi government will have the right to appoint public prosecutors, to decide land revenue matters and also to appoint or deal with electricity commission or board.

Source: http://vajiramias.com/current-affairs/governance-of-delhi-division-of- powers/5c6675f21d5def3d40d0a9ec/

Q72. The ‘Election Commission of India’ is often mentioned in the news, in this context, which of the following statements is/are not correct? a) The appointment of the chief election commissioner and other election commissioners shall be made by the president of India under Article 324(2) of the Constitution. b) The Commissioners are appointed for a 6-year period, or up to the age of 65 years, whichever is earlier. c) The Constitution has not prescribed the qualifications (legal, educational, administrative or judicial) of the members of the Election Commission. d) Since its inception in 1950, the Election Commission has been functioning as a multi-member body consisting of three election commissioners.

Answer: d

Explanation:

The appointment of the chief election commissioner and other election commissioners shall be made by the president of India under Article 324(2) of the Constitution. The Commissioners are appointed for a 6-year period, or up to the age of 65 years, whichever is earlier. The Constitution has not prescribed the qualifications (legal, educational, administrative or judicial)

of the members of the Election Commission. Since its inception in 1950 and till 15 October 1989, the election commission functioned as a single member body consisting of the Chief Election Commissioner.

On 16 October 1989, the president appointed two more election commissioners to cope with the increased work of the election commission. However, the two posts of election commissioners were abolished in January 1990 and the Election Commission was reverted to the earlier position. Again in October 1993, the president appointed two more election commissioners. Since then and till today, the Election Commission has been functioning as a multi-member body consisting of three election commissioners. Hence, option d) is the correct answer.

Source: http://vajiramias.com/current-affairs/election- commissioner/5c6675201d5def3d40d0a9e4/

Q73. Which of the statements below is correct in reference to the PCSK9 gene (proprotein convertase subtilisin/kexin type 9), recently in the news: a) It hydrolyzes orthophosphoric monoesters to alcohol and orthophosphate. b) It oxidizes long-chain fatty acids in the mitochondria. c) It is one of several evolutionarily conserved genes involved in repair of interstrand cross-links. d) It provides instructions for making a protein that helps regulate the amount of cholesterol in the bloodstream.

Answer: d

Explanation:

PCSK9 gene (proprotein convertase subtilisin/kexin type 9) provides instructions for making a protein that helps regulate the amount of cholesterol in the bloodstream. Cholesterol is a waxy, fat-like substance that is produced in the body and obtained from foods that come from animals. The PCSK9 protein appears to control the number of low-density lipoprotein receptors, which are proteins on the surface of cells. These receptors play a critical role in regulating blood cholesterol levels. In the Hyderabad edition of the TNQ’s Distinguished Lectures in Life Sciences, Geneticist Helen Hobbs told an audience of scientists shared her story of the discovery of cholesterol-lowering mutations in a human gene called PCSK9. Hence, option d) is the correct answer.

Source: http://vajiramias.com/current-affairs/pcsk9/5c63d3bf1d5def791df98669/

Q74. Consider the following statements with reference to the Contempt of court:

(1) Civil contempt refers to the willful disobedience of an order of any court.

(2) Under Article 215 High Court and Subordinate courts have special power to prosecute a person for contempt of court.

(3) Contempt of court 1971 Act was passed on the recommendations of Sanyal committee.

Which of the statements given above is/are correct? a) 1 and 2 only b) 3 only c) 1 and 3 only d) 1, 2 and 3

Answer: c

Explanation:

Statement (1) is correct: Contempt refers to the offence of showing disrespect to the dignity or authority of a court. The Act divides contempt into civil and criminal contempt. Civil contempt refers to the wilful disobedience of an order of any court. Criminal contempt includes any act or publication.

Statement (2) is incorrect: Article 215: This article explicitly gives HC the power to punish someone for their contempt. Subordinate courts have no power to prosecute a person for contempt of court.

Statement (3) is correct: Contempt of court 1971 Act was passed on recommendations of Sanyal committee. It defined what is contempt, punishment etc.

Source: http://vajiramias.com/current-affairs/contempt-of- court/5c63d2de1d5def791df98637/

Q75. Consider the following statements with reference to the National Programme on use of Space Technology:

(1) It is a proposed programme which envisages integrated use of Space and Geospatial Tools for Mapping, Monitoring and Management of Agriculture

(2) All the current running programmes, such as FASAL (for crop forecasting), NADAMS (for drought assessment), CHAMAN (for horticultural assessment and development), KISAN (for crop insurance) and Crop Intensification planning, will be subsumed under this proposed programme.

Which of the statements given above is/are correct? a) 1 only b) 2 only c) Both 1 and 2 d) Neither 1 nor 2

Answer: c

Explanation:

Statement (1) is correct: National Programme on use of Space Technology is a proposed programme which envisages integrated use of Space and Geospatial Tools for Mapping, Monitoring and Management of Agriculture.

Statement (2) is correct: All the current running programmes, such as FASAL (for crop forecasting), NADAMS (for drought assessment), CHAMAN (for horticultural assessment and development), KISAN (for crop insurance) and Crop Intensification planning, will be subsumed under this proposed programme.

Source: http://vajiramias.com/current-affairs/space-technology-in-agriculture- sector/5c63cdd51d5def791f2e1322/

Q76. “SPIN-an online platform”, recently seen in news, is related to which among the following? a) It is a platform for expediting project approval, report submission and monitoring progress of implementation of Roof Top Solar projects. b) It is an information technology platform by an e-commerce entity to act as facilitator between buyer and seller. c) It is a platform to enable safe flying of RPAS( Remotely Piloted Aerial Systems) in India. d) It is launched to help the growth, expansion and facilitation of MSME’s across the country.

Answer: a

Explanation:

Minister of State (I/C) for New and Renewable Energy and Power informed Rajya Sabha about the number of measures taken by Government to popularise Roof Top Solar Power System. List of measures taken to popularise Roof Top Solar Power System: Under the present Rooftop Solar Scheme (RTS), approved by the Government in 2015, an aggregated capacity of 2100 MW is targeted to be achieved in residential, institutional, social and Government sector through central financial assistance by the year 2019-20. Suryamitra program is being implemented for creation of a qualified technical workforce. Creation of SPIN-an online platform for expediting project approval, report submission and monitoring progress of implementation of RTS projects. Initiated Geo-tagging of RTS project, in co-ordination with ISRO, for traceability and transparency. Hence, option a) is the correct answer.

Source: http://vajiramias.com/current-affairs/roof-top-solar-power- system/5c63ced11d5def791df985ae/

Q77. With reference to “Dam Rehabilitation and Improvement Project (DRIP)”, consider the following statements:

(1) It is implemented by the Ministry of Water Resources (MoWR), Government of India, with assistance from the World Bank.

(2) It covers rehabilitation of all the large dam projects across the country.

Which of the statements given above is/are correct? a) 1 only b) 2 only c) Both 1 and 2 d) Neither 1 nor 2

Answer: a

Explanation:

Globally India ranks third after China and the USA in terms of the number of large dams with 5264 large dams in operation and 437 large dams under construction.

Statement (1) is correct: The Dam Rehabilitation and Improvement Project (DRIP) is implemented by Ministry of Water Resources (MoWR), Government of India, with assistance from World Bank with duration of 6 years (duration has been extended by 2 years).

Statement (2) is incorrect: It covers rehabilitation of 198 large dam projects located in 7 States namely Jharkhand, Karnataka, Kerala, Madhya Pradesh, Odisha, Tamil Nadu, and Uttarakhand. The overall implementation, supervision, and coordination of DRIP have been entrusted to the Central Water Commission (CWC).

Source: http://vajiramias.com/current-affairs/fifth-international-dam-safety- conference/5c63d49d1d5def79206a8bcb/ http://vajiramias.com/current-affairs/dam-rehabilitation-and-improvement-project- drip/5c18b8942099372ce74233a9/

Q78. With respect to “World Sustainable Development Summit 2019”, consider the following statements:

(1) It is hosted by the World Bank Group with an aim of expanding the scope and reach of the Summit to the global community.

(2) It started off as Delhi Sustainable Development Summit (DSDS) in 2001.

Which of the statements given above is/are correct? a) 1 only b) 2 only c) Both 1 and 2 d) Neither 1 nor 2

Answer: b

Explanation:

Statement (1) is incorrect: World Sustainable Development Summit (WSDS) is an annual event of The Energy and Resources Institute (TERI).

Statement (2) is correct: It started off as Delhi Sustainable Development Summit (DSDS) in 2001. With an aim of expanding the scope and reach of the Summit to the global community, DSDS transitioned to WSDS in 2016. The Theme of WSDS 2019 is “Attaining the 2030 Agenda: delivering on our promise”.

The CPLC Research Conference follows the 2019 edition of the World Sustainable Development Summit (WSDS) and is convened by the Carbon Pricing Leadership Coalition (CPLC). Carbon Pricing Leadership Coalition (CPLC) is hosted by the World Bank Group.

Source: http://vajiramias.com/current-affairs/world-sustainable-development-summit- 2019/5c6278d61d5def79206a6f2e/

Q79. Consider the following statements with reference to the African Union:

(1) It was formed in 2002 replacing the trading bloc Economic Community of West African States (ECOWAS).

(2) The African Continental Free Trade Agreement (AfCFTA) came into force in 2018 through the efforts of African Union.

Which of the statements given above is/are correct? a) 1 only b) 2 only c) Both 1 and 2 d) Neither 1 nor 2

Answer: d

Explanation:

Egypt’s President Abdel-Fattah el-Sissi has been elected as the chairman of the African Union at the continental body’s summit in Addis Ababa, Ethiopia.

Statement (1) is incorrect: The continental body was formed in 2002 and replaced the Organisation of African Unity, established in 1963 to end colonialism in Africa. 55 countries of the continent of Africa are its members.

Statement (2) is incorrect: African Continental Free Trade Agreement (AfCFTA) seeks to create a single market in goods and services, free movement of persons and investment, and eventually a customs union with a common external tariff. Since last year, 19 countries have signed up to the agreement and turned it into a domestic law. South Africa the continent's second largest economy, recently signed. Africa's largest economy, Nigeria is yet to sign up the agreement. But the agreement needs signatures of at least 22 countries to take effect. The decision to establish the African Continental Free Trade Area was taken as part of Africa’s efforts towards fast- tracking the continent-wide free trade component of the African Economic Community (AEC) established by the Abuja Treaty of 1991.

Source: http://vajiramias.com/current-affairs/african-union/5c6133391d5def79206a51d8/

Q80. Recently inaugurated “Titanwala Museum”, in Bagru showcases the hand-block printing of which of the following communities? a) Chhipa b) Warli c) Holeya d) Naga

Answer: a

Explanation:

Union Textiles Minister recently inaugurated the ‘Titanwala Museum’ in Bagru that showcases the Chhipa community’s hand-block printing. Bagru is a town situated 30km away from the city of Jaipur.

Bagru is well known for its handblock printing using natural dyes & techniques. This technique of hand block printing is believed to be started by the God of Chippa community “Sant Siromani Shri Namdevji Maharaj”. The Chippa’s are referred to the caste of the block printers. The traditional fabric printing and dyeing arts of Rajasthan like Bandhej, Bagru, leheriya, etc. still remain synonymous with the community. Hand block printing is a centuries old Indian art form that utilizes a hand carved teak wood block that is dipped in dye and stamped by hand onto cotton or silk. Hence, option a) is the correct answer.

Source: http://vajiramias.com/current-affairs/chhipa- community/5c74e12e1d5def712e4e5738/

Q81. With reference to “Convention on Conservation of Migratory Species (CMS)”, consider the following statements:

(1) The convention is under the aegis of International Union for Conservation of Nature.

(2) Its objective is to protect the migratory species throughout their range countries by laying down the legal foundation for internationally coordinated conservation measures.

(3) India will host the 13th Conference of Parties (COP) in 2020 at Gandhinagar in Gujarat.

Which of the statements given above are correct? a) 1 and 2 only b) 2 and 3 only c) 1 and 3 only d) 1, 2 and 3

Answer: b

Explanation:

Statement (1) is incorrect: Convention on Conservation of Migratory Species (CMS) is also referred to as the Bonn Convention. The convention has been in force, under the aegis of United Nations Environment Programme (UNEP).

Statement (2) is correct: Its objective is to protect the migratory species throughout their range countries by laying down the legal foundation for internationally coordinated conservation measures. India has been a Party to the CMS since 1983.

Statement (3) is correct: India will host the 13 th Conference of Parties (COP) of the Convention on the conservation of migratory species of wild animals (CMS) during 15th to 22nd February, 2020 at Gandhinagar in Gujarat.

Source: http://vajiramias.com/current-affairs/convention-on-the-conservation-of-migratory- species-of-wild-animals-cms/5c5e854f1d5def7a8517c850/

Q82. Boori Boot Yollo festival to welcome spring and a successful harvest is celebrated by which among the following tribes? a) Bodi, Assam b) Angami, Nagaland c) Nyishi, Arunachal Pradesh d) Garo, Meghalaya

Answer: c

Explanation:

The Nyishi tribe of Arunachal Pradesh is celebrating its fortnight-long Boori Boot Yollo festival. It is observed in the month of February every year to welcome spring and a successful harvest. Nyishi tribes, the largest community in Arunachal Pradesh, and are mainly settled in Daporijo, Upper Subansiri Locale and Raga and Dollungmukh ranges of Lower Subansiri Region of Arunachal Pradesh. Hence, option c) is the correct answer.

Source: http://vajiramias.com/current-affairs/boori-boot-yollo- festival/5c5e903e1d5def7a898273dc/

Q83. Consider the following statements with reference to the Agri-Market infrastructure fund (AMIF):

(1) It is a fund created with NITI Aayog with a corpus of 10,000 crore rupees.

(2) Its objective is to develop and upgrade agricultural marketing infrastructure in the Gramin Agricultural Markets and Agricultural Produce Market Committee.

Which of the statements given above is/are correct?

a) 1 only b) 2 only c) Both 1 and 2 d) Neither 1 nor 2

Answer: b

Explanation:

Statement (1) is incorrect: The Cabinet Committee on Economic Affairs (CCEA) has approved the creation of Agri-Market infrastructure fund (AMIF) for development and up-gradation of Gramin Agriculture Markets. A corpus of 2 thousand crore rupees will be created with NABARD.

Statement (2) is correct: The Agri-Market Infrastructure Fund was announced in 2018 Budget for developing and upgrading agricultural marketing infrastructure in the 22,000 GrAMs and 585 APMCs. It will provide the state/UT governments subsidised loans for their proposal for developing marketing infrastructure in 585 Agricultural Produce Market Committees (APMCs) and 10,000 villages and States can access AMIF for innovative integrated market infrastructure projects, including through public private partnerships.

Source: http://vajiramias.com/current-affairs/agri-market-infrastructure-fund- amif/5c5d25db1d5def7a8626e5cc/

Q84. With reference to “Dard Aryans”, consider the following statements:

(1) They are Nomadic Pastoral community of Sikkim.

(2) They are one among the list of notified Schedule Tribes.

Which of the statements given above is/are correct? a) 1 only b) 2 only c) Both 1 and 2 d) Neither 1 nor 2

Answer: d

Explanation:

Statement (1) is incorrect: Dard Aryans is a tribe of Jammu and Kashmir’s Ladakh region. The Dard Aryans inhabit Dha, Hanu, Beema, Darchik and Garkone villages in Leh and Kargil districts. The villages are together called the Aryan valley.

Statement (2) is incorrect: It is ‘NOT’ among the list of notified Schedule Tribes.

Recently Union Minister of State for Tribal Affairs informed Rajya Sabha about the steps taken by Government for Preservation of Cultural Heritage of Dard Aryan Tribe. Indira Gandhi National Centre for Arts (IGNCA) has provided Assistance in setting up a few Museums in the Regions of Dard Aryans in Leh, Laddakh, Kargil, etc. with the help of local community. A festival of Dard Aryan was organised by in January, 2019 at IGNCA, in which around 35 artists from Dard Aryan regions participated.

Source: http://vajiramias.com/current-affairs/dard-aryans/5c5d2b251d5def7a898259ae/

Q85. Consider the following statements with reference to the International Financial Services Centres Authority Bill, 2019:

(1) It will provide the establishment of a unified authority for regulating all financial services in International Financial Services Centres (IFSCs) in India

(2) The first IFSC in India has been set up at Amaravati, Andhra Pradesh.

Which of the statements given above is/are correct? a) 1 only b) 2 only c) Both 1 and 2 d) Neither 1 nor 2

Answer: a

Explanation:

Currently, the banking, capital markets and insurance sectors in IFSC are regulated by multiple regulators, i.e. RBI, SEBI and IRDAI.

Statement (1) is correct: The Union Cabinet has approved establishment of a unified authority for regulating all financial services in International Financial Services Centres (IFSCs) in India through International Financial Services Centres Authority Bill, 2019. An IFSC enables bringing back to India the financial services and transactions that are currently carried out in offshore

financial centers by Indian corporate entities and overseas branches / subsidiaries of financial institutions (FIs) by offering business and regulatory environment that is comparable to other leading international financial centers in the world like London and Singapore.

Statement (2) is incorrect: The first IFSC in India has been set up at GIFT City, Gandhinagar, Gujarat.

Source: http://vajiramias.com/current-affairs/gift-city/5c5bdf1c1d5def7a85179939/

Q86. Consider the following statements with reference to the Income-Tax Ombudsman:

(1) It was created in the year 2003 to deal with grievances of public related to settlement of complaints relating to Income Tax.

(2) Recently, Union cabinet has decided to give statutory status to the body.

Which of the statements given above is/are correct? a) 1 only b) 2 only c) Both 1 and 2 d) Neither 1 nor 2

Answer: a

Explanation:

Statement (1) is correct: The Institution of Income-Tax Ombudsman was created in the year 2003 to deal with grievances of public related to settlement of complaints relating to Income Tax.

Statement (2) is incorrect: However, the Institution of Ombudsman failed to achieve its objectives. It was observed that institution of new complaints have in turn fallen to single digits. Also, tax payers started preferring alternate methods of grievance redressal like CPGRAMS (Centralized Public Grievance Redress and Monitoring System), AaykarSevaKendras etc. further, it was also decided in 2011 to close vacant offices of Indirect Tax Ombudsman.

The Union Cabinet chaired by Prime Minister has approved the proposal for Abolition of Institution of Income-Tax Ombudsman and Indirect Tax Ombudsman.

Source: http://vajiramias.com/current-affairs/it-ombudsman/5c5bdf7c1d5def7a89823fa4/

Q87. Which of the following are the common instruments of direct democracy?

(1) Referendum

(2) Initiative

(3) Recall

(4) Presidential Form of government

Select the correct answer using the code given below: a) 1 and 2 only b) 1, 3 and 4 only c) 2, 3 and 4 only d) 1, 2 and 3 only

Answer: d

Explanation:

Recently, in Cuba, voters took part in a referendum on a new constitution that expands recognition of private property. The referendum contained only two options: yes or no to the new constitution.

The most common instruments of direct democracy are the referendum, initiative and recall.Referendum is the people’s power to approve or reject legislation. A referendum can be binding or advisory. Initiative is the power to decide on policy initiated by someone other than the legislature. Recall is the power to vote an elected official out of office. Participative democracy usually involves some measures of direct democracy within the context of an electoral democracy.

Indirect democracy, or representative democracy, is when citizens elect representatives to make laws for them. Indirect democracy too has many forms: (i) Parliamentary or Cabinet Form; (ii) Presidential Form; (iii) Unitary Form, and (iv) Federal Form. There are many mixed forms of indirect democracy. Hence, option d) is the correct answer.

Source: http://vajiramias.com/current-affairs/2019-cuban-constitutional- referendum/5c73a63e1d5def712bba6b30/

Q88. What is “Moros intrepidus”, which was in the news recently?

a) Exoplanet b) Tyrannosaur c) UNESCO heritage site d) Trans-Neptunian object located in the Kuiper belt

Answer: b

Explanation:

Scientists have discovered a new species of tiny tyrannosaur which helps explain how the dinosaurs evolved from small, speedy hunters, into the bone-crushing apex predators that we know. The species called Moros intrepidus is a small tyrannosaur who lived about 96 million years ago in the present day Utah, United States during the Cretaceous period. The tyrannosaur, described in the journal Communications Biology, is the oldest Cretaceous tyrannosaur species yet discovered in North America, narrowing a 70-million-year gap in the fossil record of dinosaurs on the continent. The bones of Moros also revealed the origin of T rex’s lineage on the North American continent. Hence, option b) is the correct answer.

Source: http://vajiramias.com/current-affairs/moros- intrepidus/5c710eef1d5def6641608441/

Q89. Consider the following statements with reference to the Asian Elephant Alliance:

(1) It is an initiative taken under the Jakarta Declaration for Asian Elephant Conservation.

(2) The goal of the alliance is to create awareness about the plight of elephants and to share knowledge and positive solutions for the better care and management of captive and wild elephants.

(3) Asian elephants listed as “endangered” in the IUCN Red List of threatened species.

Which of the statements given above is/are correct? a) 1 and 2 only b) 2 and 3 only c) 3 only d) 1, 2 and 3

Answer: c

Explanation:

Statements (1) and (2) are incorrect: Asian Elephant Alliance, an umbrella initiative by five NGOs, has come together to secure 96 out of the 101 existing corridors used by elephants across 12 States in India. The joint venture is aiming at raising £20 million (₹187.16 crore) to secure the 96 remaining elephant corridors, old and new, in the next ten years. The alliance joined hands to raise the mammoth sum as money was the main constraint in securing the land. NGOs Elephant Family, International Fund for Animal Welfare, IUCN Netherlands and World Land Trust have teamed up with Wildlife Trust of India’s (WTI) in the alliance.

Statement (1) is related to 13 countries that still have extant populations of wild Asian elephants met at the Asian Range States Meeting (AERSM) in Jakarta, Indonesia in 2017 and adopted The Jakarta Declaration for Asian Elephant Conservation.

Statement (2) is related to the goal of World Elephant Day.

Statement (3) is correct: African elephants are listed as “vulnerable” and Asian elephants as “endangered” in the IUCN Red List of threatened species.

Source: http://vajiramias.com/current-affairs/human-elephant- conflict/5c5fd8641d5def79206a36d3/ https://epaper.thehindu.com/Home/ShareArticle?OrgId=GL65CFF5K.1&imageview=0

Q90. With respect to “Know My India Programme”, consider the following statements:

(1) It is an initiative of the Ministry of External Affairs for the Persons of Indian Origin (PIO) between the age of 18 to 30 years.

(2) It is aimed to give them an exposure to various aspects of contemporary India’s forms of art, heritage and culture.

Which of the statements given above is/are correct? a) 1 only b) 2 only c) Both 1 and 2 d) Neither 1 nor 2

Answer: d

Explanation:

Statement (1) is incorrect: Know My India Programme is organized by National Foundation for Communal Harmony (NFCH) which is an autonomous organization under the administrative control of the Ministry of Home Affairs.

Recently the NFCH is organizing a special workshop for 42 Youth from 6 states in the age group of 15 to 22 years, who have been victims of communal violence in the past, under the Know My India Programme beginning in Bengaluru. The Youth come from 6 states including Jammu & Kashmir, Manipur, Assam, Chhattisgarh, Bihar and Gujarat along with 10 official mentors. This year, the programme is being organized in collaboration with the Art of Living Foundation.

Statement (2) is incorrect: Know My India Programme’s objective is to bring together children from different States and regions at one place to promote oneness, fraternity and national integration.

Know India Programme is an initiative of the Ministry of External Affairs for the Persons of Indian Origin (PIO) between the age of 18 to 30 years. It is aimed to give them an exposure to various aspects of contemporary India’s forms of art, heritage and culture.

Source: http://vajiramias.com/current-affairs/know-my-india- programme/5c57f6711d5def7a86266c68/

Q91. Consider the following statements with reference to the price monitoring and research unit (PMRU):

(1) Recently, Punjab has become the first Indian State to set up a price monitoring and research unit (PMRU).

(2) Its mandate is to track violation of prices of essential drugs and medical devices under the Drugs Price Control Order (DPCO).

Which of the statements given above is/are correct? a) 1 only b) 2 only c) Both 1 and 2 d) Neither 1 nor 2

Answer: b

Explanation:

Statement (1) is incorrect: Kerala has become the first Indian State to set up a price monitoring and research unit (PMRU).

Statement (2) is correct: Its mandate is to track violation of prices of essential drugs and medical devices under the Drugs Price Control Order (DPCO). It will offer technical help to the State Drug Controllers and the National Pharmaceutical Pricing Authority (NPPA).

The State Health Secretary would be the Chairman of the society and the Drugs Controller would be its member secretary. The move comes more than five years after the National Pharmaceutical Pricing Authority (NPPA) proposed such a system for the States and the Union Territories as there is no price control review mechanism now.

Source: http://vajiramias.com/current-affairs/price-monitoring-and-research-unit- pmru/5c568c2b1d5def7a8981cf22/

Q92. With reference to the Kanjli Wetland, consider the following statements:

(1) It is a natural wetland located on the Bein rivulet in the state of Punjab.

(2) In 2002, it was recognized by the Ramsar Convention in the List of Wetlands of International Importance.

Which of the statements given above is/are correct? a) 1 only b) 2 only c) Both 1 and 2 d) Neither 1 nor 2

Answer: b

Explanation:

Punjab state cabinet has given approval for declaring Kanjli Wetland and Kali Bein as a wildlife conservation reserve in the light of the 550th birth anniversary celebrations of Guru Nanak Dev.

Statement (1) is incorrect: Kanjli Wetland, which subsumes the Kanjli Lake, is located on the Bein rivulet in the Kapurthala district of Punjab. Kanjli Wetland is a man-made Wetland which was created in 1870 by constructing the headworks across the perennial Bien River, a tributary of the Beas River to provide irrigation facilities to the hinterland.

Statement (2) is correct: In 2002, it was recognized by the Ramsar Convention in the List of Wetlands of International Importance. The wetland also draws sacred value from the sanctity accorded to the Bein rivulet – in which Guru Nanak Dev attained enlightenment.

Source: http://vajiramias.com/current-affairs/kanjli-wetland/5c555d851d5def7a8981b7cc/

Q93. Idate Commission and Renke Commission, sometimes seen in the news, are related to: a) Protection of Western Ghats b) De-Notified tribes c) Review Company act 2013 d) Mob lynching

Answer: b

Explanation:

During the interim budget 2019-20, Government of India announced that it will set-up a Committee under Niti Aayog to complete task of identifying De-Notified, Nomadic and Semi-

Nomadic Communities not yet formally classified. A Welfare Development Board under the Ministry of Social Justice and Empowerment will be set-up for implementing welfare and development programmes for them.

The first National Commission for Denotified, Nomadic and Semi- nomadic Tribes (NCDNT) was constituted in 2003. NCDNT was reconstituted two years later under the chairpersonship of Balkrishna Renke, which submitted its report in 2008. The Renke Commission estimated their population between 10-12 crore. In 2015, Idate Commission was constituted under the chairmanship of Bhiku Ramji Idate which submitted its report in January 2018. Its key recommendations are to set up a Permanent Commission for them on the lines of similar commissions for Scheduled Castes, Scheduled Tribes, and Other Backward Classes and also set up a dedicated National Finance Development Corporation for them. Hence, option b) is the correct answer.

Source: http://vajiramias.com/current-affairs/denotified-tribes- dnt/5c5544251d5def7a8981b6af/

Q94. Consider the following statements with reference to the Motion of Thanks:

(1) It follows the address by the President to Parliament in the first session after each general election and the first session of every fiscal year.

(2) Motion of Thanks is followed by a discussion in both the houses but it is not put to vote.

(3) No amendments are allowed to be made to the Motion of Thanks.

Which of the statements given above is/are correct? a) 1 only b) 2 and 3 only c) 1 and 3 only d) 1, 2 and 3

Answer: a

Explanation:

Statement (1) is correct: The first session after each general election and the first session of every fiscal year is addressed by the president. In this address, the president outlines the policies and programmes of the government in the preceding year and ensuing year. This

address of the president, which corresponds to the ‘speech from the Throne in Britain’, is discussed in both the Houses of Parliament on a motion called the ‘Motion of Thanks’.

Statement (2) is incorrect: Article 87 provides for the special address by the President. Clause (1) of that article provides that at the commencement of the first session after each general election to the House of the People and at the commencement of the first session of each year, the President shall address both Houses of Parliament assembled together and inform Parliament of the causes of its summons. Rules were made under Clause (2) of article 87 of the Constitution to discuss the matters referred to in the President’s Address. At the end of the discussion, the motion is put to vote. This motion must be passed in the House. Otherwise, it amounts to the defeat of the government.

Statement (3) is incorrect: Notices of amendments to the Motion of Thanks on the President’s Address can be tabled after the President has delivered his Address. Amendments may refer to matters contained in the Address as well as to matters which, in the opinion of the member, the Address has failed to mention. Before 2015, there were just three occasions on which the President’s Address was amended in the Rajya Sabha, once each during the tenures of Indira Gandhi, V.P. Singh and Atal Bihari Vajpayee. In 2015 Motion of Thanks was amended on the issue of black money. In 2016 the amendments focused on legislation passed by governments in Rajasthan and Haryana limiting the rights of citizens to contest panchayat elections.

Source: http://vajiramias.com/current-affairs/presidents-address-to-joint- session/5c53e2fd1d5def7a85170427/

Q95. Consider the following statements with reference to the Thwaites Glacier:

(1) It is the first glacier of water ice on Mars detected by the NASA.

(2) It was discovered by NASA’s Operation IceBridge.

Which of the statements given above is/are correct? a) 1 only b) 2 only c) Both 1 and 2 d) Neither 1 nor 2

Answer: b

Explanation:

Statement (1) is incorrect: NASA scientists have discovered a gigantic cavity, almost 300 metres tall, growing at the bottom of the Thwaites Glacier in West Antarctica, indicating rapid decay of the ice sheet and acceleration in global sea levels due to climate change. Thwaites Glacier is an Antarctic glacier flowing into Pine Island Bay, part of the Amundsen Sea.

Statement (2) is correct:The cavity was revealed by ice-penetrating radar in NASA’s Operation IceBridge, and the findings were published in the journal Science Advances. NASA’s Operation IceBridge images Earth’s polar ice to better understand connections between polar regions and the global climate system. IceBridge studies annual changes in thickness of sea ice, glaciers and ice sheets.

It is unusually broad and fast glacier with its surface speeds exceeding 2 km/yr near its grounding line. Along with Pine Island Glacier, Thwaites Glacier has been described as part of the ‘weak underbelly’ of the West Antarctic Ice Sheet. It is predicted that it will gradually melt, leading to an irreversible collapse over the next 200 to 1000 years. Totten Glacier is a large glacier draining a major portion of the East Antarctic Ice Sheet, through the Budd Coast of Wilkes Land in the Australian Antarctic Territory.

Source: http://vajiramias.com/current-affairs/thwaites-glacier/5c53e3621d5def7a898198cf/

Q96. ‘World Survey on the Role of Women in Development’ is prepared by: a) Organization for Economic Cooperation and Development

b) Women's International League for Peace and Freedom c) Women's Refugee Commission d) UN Women

Answer: d

Explanation:

The United Nations Entity for Gender Equality and the Empowerment of Women (UN Women), is a United Nations entity working for the empowerment of women. It is a member of the United Nations Development Group. It is is empowered to support intergovernmental bodies in their formulation of policies, global standards, and norms. UN Women’s Policy and Programme Division produces a number of publications like ‘World Survey on the Role of Women in Development’ ,which comes out every five years and is presented to the Second Committee of the United Nations General Assembly, which deals with economic and financial issues. Hence, option d) is the correct answer.

Recently, UN Women has complimented the government of Odisha for proposing 33% reservation for women in Parliament and the State Assemblies. In November 2018, the Odisha Assembly passed a resolution for providing 33% reservation for women in Legislative Assemblies and Parliament.

Source: http://vajiramias.com/current-affairs/reservation-for-women-in- legislatures/5c73a96c1d5def712bba6bae/

Q97. Consider the following statements with reference to the INSTEX:

(1) It is a special purpose vehicle to facilitate trade between the China and Iran to get around US sanctions.

(2) It acts as a gold-denominated clearing house to enable trade despite US sanctions.

Which of the statements given above is/are correct? a) 1 only b) 2 only c) Both 1 and 2 d) Neither 1 nor 2

Answer: d

Explanation:

Statement (1) is incorrect: Germany, France and Britain have launched INSTEX, an EU-backed system to facilitate trade with Iran to help European businesses circumvent unilateral US sanctions on Iran. Full name is The Instrument In Support Of Trade Exchanges (INSTEX). The special purpose vehicle INSTEX aims to facilitate trade between the European Union (EU) and Iran to get around US sanctions.

Statement (2) is incorrect: Trade is initially expected to focus on non-sanctionable essential goods such as humanitarian, medical and farm products. It is not expected to address oil- related transactions, which have dropped off since last year and are Iran’s primary source of foreign currency. The three countries have sought support for the mechanism from all 28 EU member states to show European commitments in implementing the nuclear accord and to present a united front against any retaliation from Washington.

Source: http://vajiramias.com/current-affairs/instex/5c53e2831d5def7a898198be/

Q98. Consider the following statements with reference to the Rashtriya Gokul Mission (RGM):

(1) It is a project under National Programme for Bovine Breeding and Dairy Development.

(2) Its objective is to develop and conserve indigenous breeds through selective breeding.

(3) It also focuses on the issues of cattle past their reproductive or useful age.

Which of the statements given above are correct? a) 1 and 2 only b) 2 and 3 only c) 1 and 3 only d) 1, 2 and 3

Answer: a

Explanation:

Union Finance Ministry has allotted ₹302 crore to the Rashtriya Gokul Mission (RGM) for the financial year 2019-20, a reduction from last year. Rashtriya Kamdhenu Ayog will be setup for sustainable genetic up- gradation of the Cow resources. Also, a New separate Department of Fisheries for welfare of 1.5 crore fishermen will be created. Scheme is implemented on 100% grant-in-aid basis and throughout the country.

Statement (1) is correct: It is a project under National Programme for Bovine Breeding and Dairy Development.

Statement (2) is correct: Its objective is to develop and conserve indigenous breeds through selective breeding and genetically upgrading ‘nondescript’ bovine population.

Statement (3) is incorrect: It doesn’t address the issue of cattle past their reproductive or useful age.

Source: http://vajiramias.com/current-affairs/rashtriya-gokul- mission/5c555f461d5def7a85171ff1/

Q99. Which of the following pairs is/are correctly matched?

Select the correct answer using the code given below: a) 1 only b) 2 and 3 only c) 1 and 3 only d) 1, 2 and 3

Answer: a

Explanation:

Pair (1) is correctly matched: Kiru Hydroelectric Project is a run of the river scheme planned on river Chenab, a tributary of the Indus, in Kishtwar, Jammu & Kashmir. The projects would be developed by the Chenab Valley Power Projects (CVPP) -- a joint venture among National Hydroelectric Power Corporation (NHPC), state power body JKSPDC and Power Trading Corporation. It is a Concrete gravity dam.

Pair (2) is not correctly matched: The Pahari Dam Modernization Project is a water storage dam situated on Dhasan River in Jhansi district. The dam was completed in 1912 but after 100 years it is in dire need of modernisation. The Dhasan River is a right bank tributary of the Betwa River. The river originates in Raisen district in Madhya Pradesh and then forms a common boundary between M.P and Uttar Pradesh before merging into Betwa River in U.P.

Pair (3) is not correctly matched: Bhakra Dam is a concrete gravity dam on the Sutlej River in Bilaspur, Himachal Pradesh in northern India. The dam forms the Gobind Sagar reservoir.

Source: http://vajiramias.com/current-affairs/kiru-hydropower- project/5c568aaa1d5def7a8981cee8/ http://vajiramias.com/current-affairs/pahari-dam-modernization- project/5c666fdf1d5def3d40d0a987/

Q100. Consider the following statements with reference to the Stratocumulus clouds:

(1) They are the wispy and thin clouds that form at high altitudes.

(2) They are also known as ‘rain clouds’.

Which of the statements given above is/are correct?

a) 1 only b) 2 only c) Both 1 and 2 d) Neither 1 nor 2

Answer: d

Explanation:

Statement (1) is incorrect: Stratocumulus cloud consists of large, rounded masses of stratus that form groups, lines or waves. They usually have gaps between them, but they can also be joined together. Stratocumulus clouds usually form from a layer of stratus cloud breaking up. They are indicators of a change in the weather and are usually present near a warm, cold or occluded front. They are low-lying cloud.

Statement (2) is incorrect: Stratocumulus are often mistaken for rain clouds, when in reality it is quite rare to get anything more than the lightest drizzle from them, if anything at all.

Statement 1 is related to Cirrus Clouds and statement 2 is related to cumulonimbus clouds.

According to the findings published in Nature Geosciences, Stratocumulus clouds that protect us from hothouse Earth conditions by reflecting sunlight back into space could break up and vanish if carbon dioxide in the atmosphere triples. If they disappear, Earth would warm dramatically leading to melting of polar ice and lifting sea levels.

Source: http://vajiramias.com/current-affairs/stratocumulus- clouds/5c74f8b91d5def0eb883190c/